Sei sulla pagina 1di 156

1|Gener al Ma thema tics

2|Gener al Ma thema tics


(CERAE)

3|Gener al Ma thema tics


Preface

This Interactive Learning Module for Mathematics III: General Mathematics is aligned to the
competencies of the K – 12 Basic Education Curriculum set by the Department of Education. This module
is designed interactively to cater the needs and demands of the 21st century education.

In this course, as stipulated in the curriculum guide, the students must know how to solve
problems involving rational, exponential and logarithmic functions; to solve business-related problems;
and to apply logic to real-life situations.

The lessons are divided into two major components: the first quarter will focus on Functions and
Their Graphs while the second half of the quarter will focus on Basic Business Mathematics and Logic. On
the first quarter, students will be equipped with knowledge about relations, functions, evaluation of
functions, operations and compositions of functions, rational function, one-to-one functions, inverse
functions, exponential functions, and logarithmic functions. On the other hand, the second quarter of the
course will journey the learners to the world of business which includes simple interest, compound
interest, annuity, amortization, stocks and bonds, business and consumer loans and propositions and
logic.

To facilitate academic performance, Written Works (WW) are given to make sure learners can
express skills and content in written form. Performance Tasks (PT) let learners translate knowledge and
skills in diverse and real-life learning situations, finally, Quarterly Examinations (QE) will measure learning
at the end of every quarter. The components are for (WW) 25%, (PT) 50% and (QE) 25%

This Interactive Learning Module is collaboratively prepared by your Mathematics instructors


from the Ateneo de Zamboanga University – Senior High School for the school year 2020-2021.

4|Gener al Ma thema tics


TABLE OF CONTENTS

MODULE 1
AdZU Right Learning 2
AdZU SHS Delivery Framework 3
Preface 4
UNIT 1 FUNCTIONS AND THEIR GRAPHS ............................................. 6
UNIT PRE-ASSESSMENT ...................................................................... 7

LESSON 1: EVALUATION A FUNCTION ............................................... 9


LESSON 2: FUNCTIONS AS MODELS ................................................ 19
LESSON 3: OPERATIONS ON FUNCTIONS ........................................ 29
LESSON 4: RATIONAL FUNCTIONS ................................................. 39
LESSON 5: SOLVING RATIONAL EQUATION AND INEQUALITY ......... 49
LESSON 6: GRAPHING RATIONAL FUNCTIONS ................................ 60
LESSON 7: ONE-TO-ONE AND INVERSE FUNCTIONS ........................ 71
LESSON 8: REPRESENTING REAL-LIFE SITUATIONS USING
EXPONENTIAL FUNCTIONS ............................................................. 84
LESSON 9: SOLVING EXPONENTIAL FUNCTIONS, EQUATIONS, AND
INEQUALITIES .................................................................................. 95
LESSON 10: GRAPHING EXPONENTIAL FUNCTION ......................... 102
LESSON 11: THE LOGARITHMIC FUNCTION ................................... 111
LESSON 12: BASIC PROPERTIES AND LAWS OF LOGARITHMS......... 118
LESSON 13: SOLVING LOGARITHMIC FUNCTIONS, EQUATIONS, AND
INEQUALITIES ................................................................................ 126
LESSON 14: GRAPHS OF LOGARITHMIC FUNCTION........................ 133
ANSWER KEY 139
REFERENCES 152

5|Gener al Ma thema tics


UNIT 1
FUNCTIONS AND THEIR GRAPHS

6|Gener al Ma thema tics


UNIT PRE-ASSESSMENT: Let’s Check What You Know about the Unit!
Directions: Functions and Their Graphs will focus on the different types of functions and how to sketch
their respective graphs. Read each statement carefully and choose the BEST answer among the options
given. Write the letter that corresponds to your choice on the box below. Each item is worth 1 point.

#1 #2 #3 #4 #5 #6 #7 #8 #9 #10

____ ____ ____ ____ ____ ____ ____ ____ ____ ____

1. “An ADZU SHS student and his/her ID number,” what is the best relationship that best
describes them?

M. One-to-One L. One-to-Many O. Many-to-Many N. Many-to-Many

2. The following ordered pairs are considered as functions, EXCEPT.

E. {(0,1), (2, 3), (2,4), (5,6)} O. {(3,7), (4,6), (7,3), (4,5)}


I. {(−1,2), (1,2), (1, −2). (−1, −2)} A. {(3,5), (5,8), (8,3), (2,5)}

3. What type of function best represents the tricycle fare matrix in Zamboanga City?

V. Linear F. Quadratic T. Piecewise N. Rational

4. Given 𝑓(𝑥) = 2𝑥 + 1, what will the value of 𝑓(𝑥) be when 𝑥 = 1?

O. −3 D. −1 E. 1 H. 3

5. If 𝑓(𝑥) = 𝑥 + 1 and 𝑔(𝑥) = 𝑥 − 1, what is 𝑓(𝑥) + 𝑔(𝑥) equal to?

A. 2 I. 2𝑥 O. 2𝑥 + 1 U. 2𝑥 − 1

𝑥−1
6. How many solution/s does the expression = 5 have?
𝑥+1

S. One M. Two N. Three O. No solution

7. The following statements are true about one-to-one and inverse functions, EXCEPT.

B. All one-to-one functions have inverses.


S. The graph of a one-to-one function and its inverse are symmetric by the 𝑥-axis.
L. When a function’s output corresponds to a single input, the function is one-to-one.
O. All of the above.

7|Gener al Ma thema tics


1
8. Which of the following best represents the graph of 𝑓(𝑥) = ?
𝑥

I. N.

T. R.

9. What is the solution for the exponential equation 42𝑥 = 165

H. 2 A. 2.5 F. 5 R. 10

10. Logarithmic functions are often used in which of the following situations?

S. Predicting future population sizes.


E. Measuring intensity of earthquakes.
M. Finding the best-fit line for a given data set
T. Computing for the maturity value of a compound interest loan.

Bonus Question: What phrase do each correct answer from the questions form?

_____________________

Check your answers!


Scan the QR code on the right for the answer key.
You may also check the Answer Key on the last part of this unit.

8|Gener al Ma thema tics


Lesson 1 Evaluating a Function

LESSON PRE-ASSESSMENT
Directions: Evaluate the following functions correctly. Write your answer on the space provided after
each item. (15 points)

1. What is a function?
_______________________________________________________________________________
_______________________________________________________________________________

2. Determine whether each of the following is function or not.

a. 𝑓 = {(0, −1), (2, −5), (4 − 9), (6, −13)} Answer: __________________


b. 𝑔 = {(5, −10), (25, −75), (50, −100)} Answer: __________________

3. Given 𝑓(𝑥) = 𝑥 − 5, find the following values:

a. 𝑓(0) = ______________________
b. 𝑓(𝑎) = ______________________

4. Determine the domain and range of the following given functions

a. 𝑓 = {(0, −1), (2, −5), (4, −9)} Domain: ____________________


Range: _____________________

b. 𝑥 −4 −1 3 6 9 Domain: ____________________
𝑦 16 1 9 36 81 Range: _____________________

5. Which of the following graphs represent a function?

a. b.

c. d.

9|Gener al Ma thema tics


Context
Learning Competencies
At the end of the lesson, the learners can:
✓ Represent real-life situations using functions by
▪ identifying the characteristics of a function;
▪ citing an example of real-life situation using functions.
✓ Evaluate a function by
▪ identifying the values of the unknown variables;
▪ substituting the value of the unknown variables in the given function; and
▪ performing the indicated operation in the given function.

Value Integration: Competence

Experience
Prelection
Activity 1: Knowledge Web
Directions: Write five (5) words or phrases in the box that come into your mind when you encounter
the words relation and function.

RELATION

FUNCTION

Relation is a very important concept when dealing with functions. To further discuss about it, let us take
a journey on our concept notes.

10 | G e n e r a l M a t h e m a t i c s
Concept Notes
Functions
Definition Examples

RELATION - is a rule that relates 𝒇 = {(𝟏, 𝟐), (𝟐, 𝟐), (𝟑, 𝟓), (𝟒, 𝟓)}
values from set of values
𝒈 = {(𝟏, 𝟑), (𝟏, 𝟒), (𝟐, 𝟓),
(called the domain) to a
second set of values (𝟐, 𝟔), (𝟑, 𝟕)
(called the range)
𝒉 = {(𝟏, 𝟑), (𝟐, 𝟔), (𝟑, 𝟗), …,
- is a set of ordered pair
(x,y)
(𝒏, 𝟑𝒏), … }

FUNCTION - It is a correspondence 𝒇 = {(𝟏, 𝟐), (𝟐, 𝟐), (𝟑, 𝟓), (𝟒, 𝟓)}
between two sets (example
X and Y) wherein every
element in one set 𝒉 = {(𝟏, 𝟑), (𝟐, 𝟔), (𝟑, 𝟗), …,
corresponds to exactly one
element in another set. (𝒏, 𝟑𝒏), … }

- is a set of ordered pair


A B
(x,y) such that no two
ordered pairs have the
same x-value but different a 1
y-values.

b 2
-It is a relation provided
every element in domain one-to-one
(all permissible values of 𝑥 B
A
or the possible inputs)
corresponds to exactly one
element on the codomain
a 1 R
(all permissible values of 𝑦
or the possible outputs). On A
b 2
the other hand, range is N
actually the output. Not c 3 G
just the possibility of being E
d 4
the result but the result
itself. See example on the many-to-one
right.
DOMAIN CODOMAIN
-It can be one-to-one and {a,b,c,d} {1,2,3,4}
many-to-one relationship
but NOT one-to-many.

11 | G e n e r a l M a t h e m a t i c s
Examples

This is a function since each element in the


domain has a single corresponding element in
the codomain.
The relationship is one-to-one.

This is a function since each element of domain


has a single corresponding element in the
codomain, it just so happened that all of the
elements in domain has the same range.
The relationship is many-to-one.

This is not a function since the element ‘1’ in the


domain has two elements in the range: ‘0’ and
‘15’. This is still a relation but is not a function.
The relationship is one-to-many.

This is not a function because the element ‘16’


in the domain should have a corresponding
element in the codomain, which it does not
have. This is not even considered as a relation
since not all elements of domain has a single
element in the codomain.

1. A person can only have one blood type FUNCTION, ONE-TO-ONE RELATION

2. A student may only be issued one ID number FUNCTION, ONE-TO-ONE RELATION

3. Some cellular phones can have more than one sim card NON FUNCTION, ONE-TO-MANY

4. A teacher can have many students in a class NON FUNCTION, ONE-TO-MANY

12 | G e n e r a l M a t h e m a t i c s
Function as a graph in the Cartesian plane

The Vertical Line Test

A graph represents a function if and


only if each vertical line intersects
the graph ONLY once.

Example: Which of the following graphs can be graphs of functions?

a. b.

c. d.

Solution: (a) and (d) are graphs of functions while (b) and (c) are not because they do not pass the
vertical line test.

13 | G e n e r a l M a t h e m a t i c s
Evaluating Functions

Equation Values of Unknown Variable Solution

1 𝑓(−10) 𝑓(−10) =
1
substitution
𝑓(𝑥) = 6(−10)−3
6𝑥 − 3
1
𝑓(−10) = − simplify
63

𝑓(𝑥) = 5𝑥 − 3 𝑓(−1) 𝑓(−1) = 5(−1) − 3 substitution


𝑓(−1) = −8 simplify

𝑓(𝑥) = 𝑥 2 − 2𝑥 + 2 𝑓(5) a. 𝑓(5) = 52 − 2(5) + 2 substitution

b. 𝑓(5) = 25 − 10 + 2

= 17 simplify
2 𝑓(−1) 𝑓(−1) =
2
substitution
𝑓(𝑥) = 2(−1)−5
2𝑥 − 5
2
b. 𝑓(−1) =
−2−5

2
=− simplify
7

𝑓(𝑥) = 10 𝑓(−1) 𝑓(−1) = 10


= 10

Note: Evaluating a function means substituting the variable (usually 𝑥) in the function with a value from
the function’s domain and computing for the result.

14 | G e n e r a l M a t h e m a t i c s
Guided Practice
Activity 2: Eureka!
Directions: Answer the following problems by substituting the value of 𝑥 found in the first oval into the
function in the rectangle. Write the final answer in the last oval. As a challenge, try to answer the
problems within 3 minutes without a calculator. Item number 1 is done for your reference. Break a leg!

𝑓(3) = 7
𝑥=3 𝑓(𝑥) = 𝑥 + 4
1.

2𝑥 + 1
𝑓(𝑥) =
𝑥 = 1.5 𝑥−1
2.

3. 𝑥=0 𝑓(𝑥) = ξ−2𝑥 + 4

𝑥 = 2𝑥 𝑓(𝑥) = 𝑥 2 + 5𝑥 + 3
4.

1
5. 𝑥=𝑎 𝑓(𝑥) =
2

𝑥 =𝑎+𝑏 𝑓(𝑥) = 𝑥 2 + 5
6.

15 | G e n e r a l M a t h e m a t i c s
Reflection
Questions:
(1) Which activity do you like most? Why?
___________________________________________________________________________________
___________________________________________________________________________________
___________________________________________________________________________________
__________________________________________________________________________________

(2) Which of the activities do you find most challenging or least interesting and why?
___________________________________________________________________________________
___________________________________________________________________________________
___________________________________________________________________________________
__________________________________________________________________________________

(3) How did you resolve the challenges you encountered in evaluating a function? What are you
realization about the concept?
___________________________________________________________________________________
___________________________________________________________________________________
___________________________________________________________________________________
___________________________________________________________________________________

(4) In connection to our ‘knowledge web,’ what were the 3 most important words that you have learned
from this lesson? Relate them to real-life experiences.
a. ____________________________________________________________________________
____________________________________________________________________________

b. ____________________________________________________________________________
____________________________________________________________________________

c. ____________________________________________________________________________
____________________________________________________________________________

16 | G e n e r a l M a t h e m a t i c s
Action
Activity 3: Machines as Functions
Directions: A function can be represented as a machine which has an input and an output, and that the
output is related to the input by a definite rule. Your task is to think of a representation of function that
you can find in the society.

Option 1: Label what is the input, the machine (function) and the Scoring Rubric:
output by writing your answer on the figures below.
Correct Input 3 points
Option 2: Label what is the input, the machine (function) and the Correct Function 4 points
output by drawing your answer on the figures below. Correct Output 3 points
Total 10 points

Get a glimpse of the idea from the sample given below. Remember that there is a difference between
a function and a relation. Be conscious!

SAMPLE:

TEXTS/

GRAPHIC

PRINTER S ON A
PAPER
CLEAN PAPER

17 | G e n e r a l M a t h e m a t i c s
Evaluation
Test I. Evaluation of functions.
Directions: Evaluate the following functions correctly. Write your answer on the space provided after
each item. Each item is equivalent to 1 point.
1
1. Given 𝑓(𝑥) = 𝑥 2 + 3𝑥 + 6, find the following values:
2
a. 𝑓(0) = ______________________
b. 𝑓(3) = ______________________
c. 𝑓(−1) = ____________________
1
d. 𝑓 ( ) = _____________________
2

e. 𝑓(𝑎) = _____________________

2. Given 𝑓(𝑥) = −4𝑥 3 − 2𝑥 2 + 6𝑥 + 4, find the following values:


a. 𝑓(3) = ______________________
b. 𝑓(4) = ______________________
c. 𝑓(𝑥 − 3) = ___________________
d. 𝑓(12) = _____________________
e. 𝑓(10) = _____________________

2
3. Evaluate the following functions at 𝑥 =
3
a. 𝑓(𝑥) = 2𝑥 + 5

b. 𝑔(𝑥) = ξ𝑥 + 4

𝑥+5
c. 𝑔(𝑥) =
𝑥

d. 𝑚(𝑥) = −3𝑥 3 + 2𝑥 + 5

e. 𝑓(𝑥) = 10 two sets wherein

18 | G e n e r a l M a t h e m a t i c s
Lesson 2 Functions as Models
LESSON PRE-ASSESSMENT
Directions: Real-life situations may be represented as mathematical models and graphs. Below are
statements about piecewise functions. Write TRUE if the statement is correct and FALSE if otherwise.
Each item is equivalent to 1 point.

___________ 1. A function with different expression for the different parts of its domain is
considered as a piecewise function.
___________ 2. The domain of a piecewise function is always the set of real numbers.
___________ 3. Bus fare is an example of a real-life application that can be expressed as a
piecewise function.
___________ 4. In a graph of a piecewise function, hollow points may be present.
___________ 5. The graph of a piecewise function may not be continuous; breaks along the
function may be present.

Check your answers!


Scan the QR code on the right for the answer key.
You may also check the answer key on the last part of this unit.

Context
Learning Competencies
✓ Represent real-life situations using functions, including piecewise functions
✓ Solves problems involving functions.

Value Integration: Competence

19 | G e n e r a l M a t h e m a t i c s
Experience
Prelection
Activity 4: Math-ball Challenge
Directions: Various real-life situations can be represented as linear models. Below is an example of that
situation. Write a mathematical model for the given situation, construct table of values, and sketch the
graph of the mathematical model. Your answers will be graded according to the following scoring
system: Linear model – 2 points, Table of values – 2 points, Graph – 3 points

John felt hungry and went out to buy something to eat. He went to a street food stand which sells
meat balls for 2 pesos each.

Linear Mathematical Model

For the purpose of graphing, assume that the mathematical model is true for all real numbers.

Table of values Graph


𝑥 𝑦

In this activity, we were able to represent a simple scenario into a mathematical model and its graph.
In this lesson, we will be exploring other real-life situations and represent them as functions and graphs.

Concept Notes
Many real-life situations can be represented as functions. In your Junior High School years, you learned
about constant, linear, and quadratic equations and how to graph them. These equations can also be
considered as functions (i.e. constant function, linear function, and quadratic function).

According to mathisfun.com (n.d.):


“A function is a special type of relation where each input has a single output. It is often written as
𝑓(𝑥) where 𝑥 is the input value.”

20 | G e n e r a l M a t h e m a t i c s
Functions as Mathematical Models

In this lesson, we will explore different real-life scenarios that can be expressed as functions through
its mathematical equations and graphs.

In representing real-life situations are mathematical models, we consider the following process:

In representing real-life situations are mathematical models, we consider the following process:

Identify the Summarize the Write as a Answer


Assign letters
given (knowns given (tabular mathematical questions using
for the variables
and unknowns) form) model the model

To show an example, consider if the following situation gives the jeepney fare rate in a city:

A jeepney ride costs P 8.00 for the first 4 kilometers, and each additional kilometer adds
P 2.00 to the fare.

If we summarize the given in a table, we will have:


Fare cost (in pesos) Distance traveled (in km)
8 Less than or equal to 4
8 + 2*(additional kilometers) Greater than 4

Now, if we let 𝑥 be the distance traveled (in kilometers) and 𝑓(𝑥) be the fare cost (in pesos), we can
represent the given scenario as two separate functions,
𝑓(𝑥) = 8 if 𝑥 ≤ 4
𝑓(𝑥) = 8 + 2(𝑥 − 4) if 𝑥 > 4

Notice that each function has a specific domain. If we write the two functions as one (or combine them),
we will have the function,

4 𝑥≤4
𝑓(𝑥) = {
8 + 2(𝑥 − 4) 𝑥>4

The function that we came up from the given scenario is called a piecewise function.

Piecewise Functions

“A piecewise function is a function where more than one formula is used to define the output over
different pieces of the domain.” (lumenlearning.com, n.d.).

It is a conditional function that has at least two parts with a different rule for different parts of its
domain.

21 | G e n e r a l M a t h e m a t i c s
We can now use this mathematical model to jeepney fare problems. For example,

“How much will the fare cost if you travel 10 kilometers?

For this, we let 𝑥 = 10 and use 𝑓(𝑥) = 8 + 2(𝑥 − 4). That is,
𝑓(𝑥) = 8 + 2(𝑥 − 4)
𝑓(10) = 8 + 2(10 − 4)
𝑓(10) = 20 𝑝𝑒𝑠𝑜𝑠

Just like any other functions, piecewise functions can


also be graphed on a Cartesian plane. The graph for
the function model in our real-life example is given
below.

Notice that the graph for 𝑥 > 4 is a step function. This


is because of the uniqueness of the situation. For
example, if two persons traveled 4.5 kilometers and
4.9 kilometers, they will be paying the same amount
because the distance traveled is rounded up.

2𝑥 − 1 𝑖𝑓 𝑥 < 5
Another example of a piecewise function is 𝑔(𝑥) = { .
𝑥 + 2 𝑖𝑓 𝑥 ≥ 5

In graphing a piece-wise function, we separately construct the table of values for the different
expressions. For 2𝑥 − 1, we assign 𝑥-values that are less than 5 while for 𝑥 + 2, we assign 𝑥-values
greater than 5. Note that it is important to find the expression values when 𝑥 = 5. Since we are dealing
with linear expressions, we only need find two ordered pairs. That is,

2𝑥 − 1 𝑥+2
𝑥 𝑓(𝑥) 𝑥 𝑓(𝑥)

0 −1 5 7
5 9 7 9

22 | G e n e r a l M a t h e m a t i c s
It is also important to note that for the point (5, 9) will be a hollow point (not shaded) since 𝑥 = 5 is
not in the domain of 2𝑥 − 1. Hence, for the given piece-wise function, we have its graph below:

Guided Practice
Activity 5: Function Modeling and Graphing
Directions: Following the same process in the concept notes, accomplish the following items
completely.

1. Represent the given scenario as a function model. Answer the question that follows.
Function model – 7 points; Answer to the follow-up question – 3 points

A doctor’s consultation fee is based on the length of time. He charges P100 for
the first 6 minutes, P200 for over 6 minutes to 15 minutes, and P200 plus P10
per minute for over 15 minutes. How much will the consultation fee be for 30
minutes?

Assignment of variables:

Function model:

23 | G e n e r a l M a t h e m a t i c s
Answer to the question (solution):

2. Construct the table of values for each expression and sketch the graph of the given piecewise
function below.
Table of values – 6 points; Graph – 6 points

𝑥+4 𝑥 ≤ −2
𝑓(𝑥) = { 3𝑥 −2 <𝑥 ≤ 3
2+𝑥 𝑥>3

𝑥 𝑓(𝑥)

𝑥 𝑓(𝑥)

𝑥 𝑓(𝑥)

24 | G e n e r a l M a t h e m a t i c s
Reflection
Questions:
1. What was your most dominant feeling as you were accomplishing the guided practice?
________________________________________________________________________________
________________________________________________________________________________
________________________________________________________________________________
________________________________________________________________________________

2. In answering the two items, which part of the process did you find the most difficult? Why do you
say so?
________________________________________________________________________________
________________________________________________________________________________
________________________________________________________________________________
________________________________________________________________________________

3. What do you think were the factors why these parts were difficult for you?
________________________________________________________________________________
________________________________________________________________________________
________________________________________________________________________________
________________________________________________________________________________

4. What realizations did you have on this lesson? How will these realizations help you improve as a
student?
________________________________________________________________________________
________________________________________________________________________________
________________________________________________________________________________
________________________________________________________________________________
________________________________________________________________________________
________________________________________________________________________________

5. How can you sharpen your skills in solving problems involving functions for you to be more
academically competent in the field of mathematics?
________________________________________________________________________________
________________________________________________________________________________
________________________________________________________________________________
________________________________________________________________________________
________________________________________________________________________________
________________________________________________________________________________

25 | G e n e r a l M a t h e m a t i c s
Action
Activity 6: Let’s Talk About Functions
Directions: In this lesson, you learned that real-life situations can be represented as
functions. Aside from the situations presented in this lesson, think of other real-life
situations representing piecewise functions. Identify one question that can be answered
using the data given in the situation. Choose one of the following tasks for your output and
use the space provided below for your output.

Option 1: Create a short comic strip that describes and explains the situation. Include the
possible ways on how to solve the problem using the concept of functions.

Option 2: Create a text/chat conversation that describes and explains the situation. Include
the possible ways on how to solve the problem using the concept of functions.

Your output will be graded according using the scoring rubric below:

CRITERIA 4 3 2 1
Content All messages 1-2 sentences do 3-4 sentences do There are no
involve the concept not involve the not involve the concepts on
(x3) of functions. concept of concept of functions used.
functions. functions.

Dialogue There are 5-6 There are only 3-4 There are 2 There is only one
exchange of exchange of exchange of exchange of
(x2) information information information information
present. present present. present.

Conventions All sentences are 1-2 sentences have 3-4 sentences have More than 4
(x1) grammatically grammatical errors. grammatical errors. sentences have
correct. There are There are a few There are some grammatical errors.
no misspelled misspelled words. misspelled words. There are many
words. misspelled words.

TOTAL

26 | G e n e r a l M a t h e m a t i c s
Evaluation
Test I. Functions as Models.
Directions: Real-life situations can represent piecewise functions. The following item is a real-life
situation of a piecewise function. Write a piecewise function that models the situation and answer the
question that follows. Write your answers and solutions on the boxes provided below. This item is worth
10 points: Function Model – 7 points, Solution to the question – 3 points

1. As of writing, Zamboanga City is under General Community Quarantine (GCQ). With this, public
utility vehicles such as tricycles are now allowed to operate but with restrictions. Tricycle fares
are charged P20 for the first kilometer and additional P5 for every additional kilometer.

Piecewise Function Model:

How much should a passenger pay if she travels from KCC to her house that is 5 kilometers from
KCC? (3 points)

27 | G e n e r a l M a t h e m a t i c s
Test II. Graphing Piece Functions.
Directions: Like any other functions, piecewise functions can also be represented through its
graph. Given the following piecewise function, construct the table of values for each condition
and sketch the graph. Use the tables and Cartesian plane provided below. This item is worth
10 points: table of values – 5 points; Graph – 5 points

10𝑥 − 7 𝑥>1
𝑓(𝑥 ) = {3𝑥 + 2 𝑥<1
𝑥2 𝑥=1
𝑥 𝑓(𝑥) 𝑥 𝑓(𝑥) 𝑥 𝑓(𝑥)

28 | G e n e r a l M a t h e m a t i c s
Lesson 3 Operations on Functions
LESSON PRE-ASSESSMENT
Directions: Solve the following problems completely by applying the indicated operation. Write your
answer on the space provided after each item. Each item is equivalent to 3 points.

If 𝑓(𝑥) = 2𝑥 + 3 and 𝑔(𝑥) = −4𝑥 − 5, find

a. (𝑓 + 𝑔)(𝑥) b. (𝑓 − 𝑔)(𝑥)

c. (𝑓 ∙ 𝑔)(𝑥) d. (𝑓/𝑔)(𝑥)

e. (𝑓 ∘ 𝑔)(𝑥) f. (𝑔 ∘ 𝑓)(𝑥)

29 | G e n e r a l M a t h e m a t i c s
Context
Learning Competencies
At the end of the lesson, the students can:
✓ perform addition, subtraction, multiplication, division, and composition of functions:
▪ identifying the indicated operation on the problem;
▪ using properties and definitions in computing for the answer; and
▪ simplifying the answer.

Value Integration: Competence

30 | G e n e r a l M a t h e m a t i c s
Experience
Prelection
Activity 7: Bring me Back with Ease!
Direction: Read the skit below and answer the problem given. Fighting!
AHA! Do you still remember how to count from 1 to 10?

Of course! Are you still familiar with the basic operations like
addition, subtraction, multiplication and….

Division! Oh yeah! I still do! Those were our glimpse of first grade!

Ohhhh. If you and I can remember those things, then YOU (Yes
you, the one who is reading) ALSO HAVE THOSE MEMORIES
WITH YOU!
Let’s see! Why don’t you try to solve the problem below?

That is amazing! All you have to do is to pick a number and operation


on the box then fill in the line given. The challenge is for you to
combine those numbers and operations that will result to 1.

Yes! One. Your goal is to have a correct equation! That’s right!

Best of luck! You can do it!

9 ( − = 𝟏

31 | G e n e r a l M a t h e m a t i c s
Concept Notes

SUM, DIFFERENCE, PRODUCT, and QUOTIENT of a FUNCTION


Given two functions 𝑓(𝑥) and 𝑔(𝑥), we define the following:
SUM
(𝒇 + 𝒈)(𝒙) = 𝒇(𝒙) + 𝒈(𝒙)

Given: If 𝑓(𝑥) = 6𝑥 2 − 12𝑥 − 19 and 𝑔(𝑥) = 𝑥 3 − 𝑥 2 + 8𝑥 − 1, find (𝑓 + 𝑔)(𝑥).

Solution:

(𝑓 + 𝑔)(𝑥) = 𝑓(𝑥) + 𝑔(𝑥) Equivalent equation or Distribute


Property

(𝑓 + 𝑔)(𝑥) = (6𝑥2 − 12𝑥 − 19) + (𝑥3 − 𝑥2 + 8𝑥 − 1) Substitution

(𝑓 + 𝑔)(𝑥) = 𝑥3 + (6𝑥2 − 𝑥2 ) + (−12𝑥 + 8𝑥) + (−19 − 1) Grouping similar terms and


arranging in descending order

(𝑓 + 𝑔)(𝑥) = 𝑥3 +5𝑥2 − 4𝑥 − 20 Simplifying


DIFFERENCE
(𝒇 − 𝒈)(𝒙) = 𝒇(𝒙) − 𝒈(𝒙)

Given: If 𝑓(𝑥) = 6𝑥 2 − 12𝑥 − 19 and 𝑔(𝑥) = 𝑥 3 − 𝑥 2 + 8𝑥 − 1, find (𝑓 − 𝑔)(𝑥)

Solution:

(𝑓 − 𝑔)(𝑥) = 𝑓(𝑥) − 𝑔(𝑥) Equivalent equation or Distribute


Property

(𝑓 − 𝑔)(𝑥) = (6𝑥2 − 12𝑥 − 19) − (𝑥3 − 𝑥2 + 8𝑥 − 1) Substitution

(𝑓 + 𝑔)(𝑥) = −𝑥3 + (6𝑥2 + 𝑥2 ) + (−12𝑥 − 8𝑥) + (−19 + 1) Grouping similar terms and
arranging in descending order

(𝑓 + 𝑔)(𝑥) = −𝑥3 + 7𝑥2 − 20𝑥 − 18 Simplifying


PRODUCT
(𝒇 ∙ 𝒈)(𝒙) = 𝒇(𝒙) ∙ 𝒈(𝒙)

Given: If 𝑓(𝑥) = 𝑥 3 − 27 and ℎ(𝑥) = 4𝑥 − 1, 𝑓𝑖𝑛𝑑 (𝑓 ∙ ℎ)(𝑥)

Solution:

(𝑓. ℎ)(𝑥) = 𝑓(𝑥) ∙ ℎ(𝑥) Equivalent equation

(𝑓. ℎ)(𝑥) = (𝑥3 − 27)( 4𝑥 − 1) Substitution

(𝑓. ℎ)(𝑥) = 4𝑥4 − 𝑥3 − 108𝑥 + 27 Distribute Property

32 | G e n e r a l M a t h e m a t i c s
QUOTIENT
𝒇 𝒇(𝒙)
( ) (𝒙 ) = , 𝒈(𝒙) ≠ 𝟎
𝒈 𝒈(𝒙)
𝑓
Given: If 𝑓(𝑥) = 𝑥 3 − 27 𝑎𝑛𝑑 𝑔(𝑥) = 𝑥 2 + 3𝑥 + 9 , find ( ) (𝑥)
𝑔

Solution:
𝑓 𝑓(𝑥) Equivalent equation
( ) (𝑥) =
𝑔 𝑔(𝑥)
𝑓 𝑥3 − 27 Substitution
( ) (𝑥) =
𝑔 𝑥2 + 3𝑥 + 9
𝑓 (𝑥 − 3)(𝑥2 + 3𝑥 + 9) Factor the numerator
( ) (𝑥) =
𝑔 𝑥2 + 3𝑥 + 9
𝑓 Simplifying
( ) (𝑥) = 𝑥 − 3
𝑔

Composition of Functions
Given two functions 𝑓(𝑥) and 𝑔(𝑥), the composite function denoted by (𝑓 ∘ 𝑔)(𝑥) is defined by
(𝑓 ∘ 𝑔)(𝑥) = 𝑓(𝑔(𝑥))
and the domain of (𝑓 ∘ 𝑔)(𝑥) is the set of all numbers 𝑥 in the domain of 𝑔(𝑥), such that 𝑔(𝑥) is the domain of
𝑓(𝑥).
Similarly, we define (𝑔 ∘ 𝑓)(𝑥) as follows:
(𝑔 ∘ 𝑓)(𝑥) = 𝑔(𝑓(𝑥))
Given: If 𝑓(𝑥) = 𝑥 2 − 6 and 𝑔(𝑥) = 3𝑥 + 2, find
1. (𝑓 ∘ 𝑔)(𝑥) 3. (𝑔 ∘ 𝑔)(𝑥)
2. (𝑔 ∘ 𝑓)(𝑥) 4. (𝑓 ∘ 𝑓)(𝑥)
Solutions:

(𝑓 ∘ 𝑔)(𝑥)

(𝑓 ∘ 𝑔)(𝑥) = 𝑓(𝑔(𝑥)) Definition of composition of functions


(𝑓 ∘ 𝑔)(𝑥) = 𝑓(3𝑥 + 2)
(𝑓 ∘ 𝑔)(𝑥) = (3𝑥 + 2)2 − 6 Apply the definition of f
(substitute the value of 𝑔(𝑥) into the 𝑥 of 𝑓(𝑥))

(𝑓 ∘ 𝑔)(𝑥) = 9𝑥2 + 12𝑥 + 4 − 6 Expand (3𝑥 + 2)2 using distributive property

(𝑓 ∘ 𝑔)(𝑥) = 9𝑥2 + 12𝑥 − 2 Simplifying the equation

33 | G e n e r a l M a t h e m a t i c s
(𝑔 ∘ 𝑓)(𝑥)
(𝑔 ∘ 𝑓)(𝑥) = 𝑔(𝑓(𝑥)) Definition of composition of functions
(𝑔 ∘ 𝑓)(𝑥) = 𝑔(𝑥2 − 6)

(𝑔 ∘ 𝑓)(𝑥) = 3(𝑥2 − 6) + 2 Apply the definition of g


(substitute the value of 𝑓(𝑥) into the 𝑥 of 𝑔(𝑥)).

(𝑔 ∘ 𝑓)(𝑥) = 3𝑥2 − 18 + 2 3(𝑥 2 − 6). Multiply 3 with the values inside the parenthesis.

(𝑔 ∘ 𝑓)(𝑥) = 3𝑥2 − 16 Simplifying

(𝑔 ∘ 𝑔)(𝑥)
(𝑔 ∘ 𝑔)(𝑥) = 𝑔(𝑔(𝑥)) Definition of composition of functions
(𝑔 ∘ 𝑔)(𝑥) = 𝑔(3𝑥 + 2)
(𝑔 ∘ 𝑔)(𝑥) = 3(3𝑥 + 2) + 2 Apply the definition of g
(substitute the value of 𝑔(𝑥) into the 𝑥 of 𝑔(𝑥))

(𝑔 ∘ 𝑔)(𝑥) = 9𝑥 + 6 + 2 3(3𝑥 + 2). Multiply 3 into the values inside the parenthesis.
(𝑔 ∘ 𝑔)(𝑥) = 9𝑥 + 8 Simplifying

(𝑓 ∘ 𝑓)(𝑥)
(𝑓 ∘ 𝑓)(𝑥) = 𝑓(𝑓(𝑥)) Definition of composition of functions
(𝑓 ∘ 𝑓)(𝑥) = 𝑓(𝑥2 − 6)
2 Apply the definition of f
(𝑓 ∘ 𝑓)(𝑥) = (𝑥2 − 6) − 6
(substitute the value of 𝑓(𝑥) into the 𝑥 of 𝑓(𝑥))

(𝑓 ∘ 𝑓)(𝑥) = 𝑥4 − 12𝑥2 + 36 − 6 2
Expand (𝑥2 − 6) using distributive property

(𝑓 ∘ 𝑓)(𝑥) = 𝑥4 − 12𝑥2 + 30 Simplifying

34 | G e n e r a l M a t h e m a t i c s
Guided Practice
Activity 8: Simplified Combo!
Direction: Solve each item completely by applying the indicated operations. The functions are found in
the first two boxes found on top. Fill in the bigger boxes your answers. Start your timer and make it
happen in 10 minutes! Good luck! Each item is equivalent to 2 points.

𝑓(𝑥) = 3𝑥 − 2 𝑔(𝑥) = 𝑥 2

(𝑓 + 𝑔)(𝑥) (𝑓 − 𝑔)(𝑥)

(𝑓 ∙ 𝑔)(𝑥) (𝑓/𝑔)(𝑥)

(𝑓 ∘ 𝑔)(𝑥) (𝑔 ∘ 𝑓)(𝑥)

(𝑓 ∘ 𝑓)(𝑥) (𝑔 ∘ 𝑔)(𝑥)

35 | G e n e r a l M a t h e m a t i c s
Reflection
Questions:
1. Which of the activity did you like the least and why?
___________________________________________________________________________________
___________________________________________________________________________________
___________________________________________________________________

2. Which among the activities did you enjoy the most and why?
___________________________________________________________________________________
___________________________________________________________________________________
___________________________________________________________________

3. What are your learnings from doing the activity?


___________________________________________________________________________________
___________________________________________________________________________________
___________________________________________________________________________________
___________________________________________________________________________________
___________________________________________________________________________________
___________________________________________________
4. How do you use the operations of functions in real-life to attain your desired goal?
___________________________________________________________________________________
___________________________________________________________________________________
___________________________________________________________________________________
___________________________________________________________________________________
___________________________________________________________________________________
___________________________________________________

36 | G e n e r a l M a t h e m a t i c s
Action
Activity 9: Math in my Life!
Directions: Operations on functions help us realize that overlapping domains can be combined using
the proper operation. Now, your task is to fill the boxes below with a scenario or a situation in your life
that you think would apply the concept of operations on functions.

Option 1: You can describe the situation with at most 2 sentences.


Option 2: You can draw the situation to help the reader imagine it vividly.

Wonder. It empowers your thinking and imagination.


Realize. It helps you see things differently.
Appreciate. For with slightest appreciation comes the purest joy and love.
MATH can be challenging but it can also be meaningful. Try it!

O Now that I graduated from Junior High


P School. I realized that overlapping values
E of determination and hard work can be
R multiplied in order for me to achieve my
A dreams.
T
I
O
N
S

I
N

M
Y

L
I
F
E

37 | G e n e r a l M a t h e m a t i c s
Evaluation
Test I. Operations of Functions.
Directions: Solve the problems completely. Each item is equivalent to 3 points.

1. If 𝑓(𝑥) = 2𝑥 − 1and 𝑔(𝑥) = 𝑥 3 − 216, find


a. (𝑓. 𝑔)(𝑥)

b. (𝑔 ∘ 𝑓)(𝑥)

c. (𝑓 ∘ 𝑓)(𝑥)

2. If 𝑚(𝑥) = 3𝑥 2 + 18𝑥 + 27 and 𝑛(𝑥) = 𝑥 − 6, find


𝑚
a. ( )(𝑥)
𝑛

b. (𝑛 ∘ 𝑛)(𝑥)

38 | G e n e r a l M a t h e m a t i c s
Lesson 4 Rational Functions
LESSON PRE-ASSESSMENT
Directions: Read and understand the following problems carefully. Write a model that best represents
the given situation. Write all your answers in the box provided for each item.

1. Joven and Ritchie work for a book printing company. Joven can print 5 books in 8 hours. Together
they can do the same job in 5 hours.

2. Jenny walks 5 kilometers from her house to Southway Square to buy a new bike which she uses
to return home. She averaged 10 kilometers faster on her bike than on foot. Hint: Use the
𝑑
formula 𝑣 = .
𝑡

3. When five times the square of a certain integer is divided by four more than three times the
same number, the result is −10. Form an equation that models the given situation.

4. A domestic ferry traveled for 222 km when it moved along with the current, while it traveled 66
km when it moved against the current in the same amount of time.

39 | G e n e r a l M a t h e m a t i c s
Context
Learning Competencies
At the end of the unit, the learners can:
✓ represent real-life situations using rational functions and equations by:
▪ identifying the characteristics of rational function; and
▪ giving examples of rational function.

Value Integration: Competence

40 | G e n e r a l M a t h e m a t i c s
Experience
Prelection
Activity 10
Directions: In your early mathematics, the concept of fractions, ratios and proportion were already
introduced. Now, try to recall the ways on how to represent ratios and proportion by giving examples
then write it on the notes given below.

41 | G e n e r a l M a t h e m a t i c s
Concept Notes

Rational Equation Rational Inequality Rational Function

An equation (with An inequality (inequality A function of the form of 𝑓(𝑥) =


equal sign) involving symbol such as <, >, ≤ 𝑝(𝑥) where 𝑝(𝑥)𝑎𝑛𝑑 𝑞(𝑥) are
Definition rational expressions. , ≥) involving rational 𝑞(𝑥)
expressions. polynomials, and 𝑞(𝑥) is not the
zero function or equal to zero.
𝑥 2 + 2𝑥 + 3
𝑓(𝑥) =
2 3 1 𝑥+1
5 2 or,
Example − = ≤
𝑥 2𝑥 5 𝑥−3 𝑥 𝑥 2 + 2𝑥 + 3
𝑦=
𝑥+1

Representation of Rational Functions

Example 1

d
Average speed (or velocity) can be computed by the formula s = . Consider a 100-meter
t
track used for foot races. The speed of a runner can be computed by taking the time it will take
100
him to run the track and applying it to the formula s = , since the distance is fixed at 100
t
meters. Represent the speed of a runner as a function of the time it takes to run 100 meters in
the track.

Solution
Since the speed of a runner depends on the time it takes the runner to run 100 meters, we can
represent speed as a function of time.

Let 𝑥 represent the time it takes the runner to run 100 meters.
𝟏𝟎𝟎
Then the speed can be represented as a function s(x) as follows: 𝒔(𝒙) =
𝒙
𝑑
*Note: Observe that it is similar in structure to the formula 𝑠 = relating speed, distance and time.
𝑡

Following a systematic approach, we come up with a simplified procedure to solve similar problems.

42 | G e n e r a l M a t h e m a t i c s
Example 2
A company manufacturing hats has daily fixed costs of Php 30,000 and total costs of Php 51,000 for a
daily output of 200 hats. Assume that the total cost per day is linearly related. Write a function
representing average daily cost per hat.

Solution

Step 1: Recall that the equation of the cost function is defined by 𝐶(𝑥) = 𝑚𝑥 + 𝑏, where 𝑚 is the variable
cost and 𝑏 is the fixed cost. From the problem, we have the following given:

𝑏 = 𝑃ℎ𝑝30,000 𝐶 (200) = 𝑃ℎ𝑝51,000 𝑥 = 200

Step 2: Note that, here’s a given fixed cost but no variable cost, so we have to find m by using the formula,

𝐶(𝑥) = 𝑚𝑥 + 𝑏
𝐶(200) = 𝑚(200) + 30000
51000 = 200𝑚 + 30000
21000 = 200𝑚
𝑚 = 105
𝐶(𝑥)
The cost function is 𝐶(𝑥) = 105𝑥 + 30000. If 𝐶(𝑥) is the total cost, then average cost is .
𝑥
Therefore, the average cost function is

105𝑥 + 30000
𝐶(𝑥) =
𝑥

Representation of Rational Equation

Example 3:
In an inter-barangay basketball league, the team from Barangay Culiat has won 12 out games of
25 games, a winning percentage of 48%. Write an equation that models the given situation.

Solution:

Step 1: Let 𝑥 represent the number of games that they need to win to raise their percentage to 60%.
The team has already won 12 out of their 25 games. If they win 𝑥 games in a row to increase their
percentage to 60%, then they would have played 12 + 𝑥 games out 25 + 𝑥 games.

12+𝑥
Step 2: Writing the model, we have the equation, = 0.6
25+𝑥 Recall, conversion of
percentage to decimal
form form

43 | G e n e r a l M a t h e m a t i c s
Example 4:
63
The difference of a whole number and its reciprocal is . Write an equation that models the
8
problem.

Solution:

1
Step 1: Let us represent the unknown in the given problem. Let x be the whole number and be the
𝑥
63
reciprocal of that number. Note that the difference is equal to .
8
Step 2: From these, we form the equation that describes the problem. we now have the following
equation,
1 63
𝑥− =
𝑥 8

Guided Practice
Direction: Read and understand the given problem. Then write a representation that models the given
problem. Show a complete solution.

1. A charitable institution has generated Php20,000.00 in equal donations from their members
to share the cost of decorating materials for their new office.

Step 1 Step 2 Answer

44 | G e n e r a l M a t h e m a t i c s
2. A certain boat travelled 32 kilometers along with the current and 16 kilometers against the
current. In the same amount of time, find the rate of the boat if the rate of the current is 2kph.

Step 1 Step 2 Answer

33
3. Twice the sum of a number and its reciprocal is same as .
16

Step 1 Step 2 Answer

45 | G e n e r a l M a t h e m a t i c s
Reflection
Questions:
1. How did you find the activity?

________________________________________________________________________________
________________________________________________________________________________
_________________________________________________________________________.

2. What part in the activity challenges you the most? Why?

________________________________________________________________________________
________________________________________________________________________________
_________________________________________________________________________.

3. How did you resolve the challenges you encountered?

________________________________________________________________________________
________________________________________________________________________________
_________________________________________________________________________.

4. What are the things you gained that highlight your learnings?

a. __________________________________________________________________________.
b. __________________________________________________________________________.
c. __________________________________________________________________________.

46 | G e n e r a l M a t h e m a t i c s
Action
Activity 11: Now What?
Direction: Most real-life problems can be represented and solved by applying the concept of rational
functions as discussed previously. Now it is your turn to observe, look around, and create a situation
on your own which you think is related to the topic. Also, write a model that best describes it on the
space provided.

Reminders:
*Kindly exclude the number-related problem
*Make sure not to copy the problems above by just changing the given values
*As much as possible make it unique

47 | G e n e r a l M a t h e m a t i c s
Evaluation
Directions: Read and understand the following problem. Write a model which best describes the given
situation. Show your complete solution on the space provided.

1. The secretary of the council can draft a communication letter in 15 minutes while the assistant
secretary can finish the same in 32 minutes.

3
2. A number added to four times its reciprocal is equal to the sum of and the quotient of two or
2
more than three times the same number and four.

3. Celestine rides her power boat up and down the Merloquet river. The water in the river flows at 6
miles per hour. Celestine then takes 5 hours longer to travel 360 miles against the current than she
does to travel 360 miles along with the current. Write a model that shows how to get the speed of
Celestine’s boat in still water.

4. Leticia got 80%, 65% and 70% on his three long exams in Mathematics. For her to pass this grade’s
component, she must obtain an average of at least 75% in all his long exams. How much should she
obtain on the last long exam to earn at least 75%?

48 | G e n e r a l M a t h e m a t i c s
Lesson 5 Solving Rational Equation and Inequality
LESSON PRE-ASSESSMENT
Directions: Match each item in column I that corresponds to your answer found in column II. Then on
column III, identify whether the given item in column I is a rational function, rational equation or
rational inequality. Write your answer on the space provided.

I II III

___
x 5x 1
+ = a.)
 x   x  0 _____________________
2 3 6
2 3 1 1 1
___ − = b.) (−∞, −1)⋃ [− , 0) ⋃[ , ∞))
2 2
_____________________
x 2x 5
3 1
___ + 4 c.) x = 13 _____________________
x +1 x
3
___ c( x) = d.) x = 2.5 _____________________
x
e.) x = 1.3

Context
Learning Competencies
At the end of the unit, the students can:
✓ distinguish rational function, rational equation, and rational inequality by:
▪ identifying the characteristics of rational function, rational equation and rational
inequality
▪ giving examples of rational function, rational equation and rational inequality
✓ solve rational equations and inequalities by:
▪ identifying the characteristics of a rational equation and rational inequalities
▪ identifying procedures in solving rational equation and inequalities.

Value Integration: Competence

49 | G e n e r a l M a t h e m a t i c s
Experience
Prelection
Activity 12
Directions: Find the least common denominator or LCD of the following expression. Write your answer
on the line below each box.

6 4 3 2 3 2 x
+ + + x + −5
x+2 x−2 2 5x 2 3x + 2 4

________________ _____________ ________________ _________________

Directions: Factor the following completely.

4 x 2 − 25 5a − 20 x2 + 2 x − 3 x2 + 2 x − 3

_____________ _____________ _______________ _______________

Concept Notes
Procedure for Solving Rational Equations
To solve rational equations:
a. Eliminate denominators by multiplying each term of the equation by the least common
denominator.
b. Note that eliminating denominators may introduce extraneous solutions. Check the solutions of
the transformed equation with the original equation.

Example 1: Solve for 𝑥.


𝑥 1 8
− =
𝑥+2 𝑥−2 𝑥2 −4
Solution Explanation
𝑥 1 8 Factor each denominator in the
− = fraction.
𝑥+2 𝑥−2 (𝑥 + 2)(𝑥 − 2) Add link
The LCD is (𝑥 + 2)(𝑥 − 2).
𝑥 1 8 Multiply both sides of the equation
(𝑥 + 2)(𝑥 − 2)( ) − (𝑥 + 2)(𝑥 − 2)( ) = (𝑥 + 2)(𝑥 − 2)( )
𝑥+2 𝑥−2 (𝑥 + 2)(𝑥 − 2)
by the LCD.

50 | G e n e r a l M a t h e m a t i c s
Simplify
𝑥 (𝑥 + 2)(𝑥 − 2)(𝑥)
(𝑥 + 2)(𝑥 − 2) ( )=
𝑥+2 𝑥+2
= (𝑥 + 2)𝑥.

1 (𝑥 + 2)(𝑥 − 2)(1)
(𝑥 + 2)(𝑥 − 2) ( )=
𝑥−2 𝑥−2
(𝑥 − 2)𝑥 − (𝑥 + 2) = 8 = (𝑥 − 2).

8
(𝑥 + 2)(𝑥 − 2) ( )
(𝑥 + 2)(𝑥 − 2)
(𝑥 + 2)(𝑥 − 2)(8)
= = 8.
(𝑥 + 2)(𝑥 − 2)

Simplify
(𝑥 − 2)𝑥 = 𝑥2 − 2𝑥.
Thus,
𝑥2 − 3𝑥 − 10 = 0 𝑥2 − 2𝑥 − (𝑥 + 2) = 8
2
𝑥 − 2𝑥 − 𝑥 − 2 = 8
𝑥2 − 2𝑥 − 𝑥 − 2 − 8 = 0
𝑥2 − 3𝑥 − 10 = 0
(𝑥 + 2)(𝑥 − 5) = 0 Factoring
Simplify
(i) 𝑥+2 = 0
𝑥 = −2
𝑥 = −2 𝑜𝑟 𝑥 = 5
(ii) 𝑥−5 = 0
𝑥=5
𝑥 1 8
Checking: − =
𝑥+2 𝑥−2 𝑥2 −4

Solution Explanation
If 𝑥 = −2
−2 1 8 Substitute −2 into the x of the
− =
−2 + 2 −2 − 2 (−2)2 − 4 equation.
−2 1 8
− = Simplify
0 −4 0
Since 𝑥 = −2 makes the equation
Undefined undefined, then it is an extraneous
solution.

Solution Explanation
If 𝑥 = 5
5 1 8 Substitute 5 into the x of the
− = 2 equation.
5+2 5−2 (5) − 4

51 | G e n e r a l M a t h e m a t i c s
5 1 8 Simplify
− = 5 5
7 3 21 =
5+2 7

1 1
=
5−2 3

8 8
=
(5)2 − 4 21

8 8 Since 𝑥 = 5 satisfies the original


=
21 21 equation, it is the only solution.

Procedure for Solving Rational Inequality


Rational inequalities are solved in almost the same way as polynomial inequalities. The process differs
only because rational inequalities contain denominators which may have some values, where they are
not defined. Thus, solving rational inequalities requires extra care.
a. Find the zeroes of numerator.
b. Find the restricted values for the denominator (the values of 𝑥 that make the denominator 0).
c. Divide the number line into intervals using the zeroes and the restrictions of the denominator as
partition points.
d. Determine the sign of the given rational inequality on each interval. Use test points whenever
necessary. Rewrite the given inequality into general form (isolating all terms on one side).
State the solution set. Refer to the original rational inequality for the appropriate interval notations to
use.

2𝑥−3
Example 1: Consider the rational inequality ≥0.
𝑥+2
Step 1: Find the zeroes of the numerator.
Solution Explanation
2𝑥 − 3 = 0 Numerator
3 Simplify
𝑥=
2 2𝑥 − 3 = 0
2𝑥 = 3
3
𝑥=
2

Step 2: Find the restrictions for the denominator.


Solution Explanation
𝑥+2=0 Denominator
𝑥 = −2 Simplify

52 | G e n e r a l M a t h e m a t i c s
Step 3: Divide the number line into intervals using 𝟑/𝟐 and −𝟐 as partition points.

− 

𝟑 𝟑
The intervals are (−∞, −𝟐), (−𝟐, ) 𝒂𝒏𝒅 ( . ∞).
𝟐 𝟐

Step 4: Determine the sign of the rational inequality at each interval.

Factor/Intervals (−∞, −𝟐) 𝟑 𝟑 −𝟐 𝟑


(−𝟐. ) ( , ∞)
𝟐 𝟐 𝟐
𝟐𝒙 − 𝟑 − − + − 0
𝒙+𝟐 − + + 0 7
2
𝟐𝒙 − 𝟑 + − + UNDEFINED 𝟎
𝒙+𝟐

Get any value between the intervals and substitute it on the given factor then get the sign of the
result.
Example:
−5 is an element of (−∞, −2). So, substituting it on the factor we have, 2(−5) − 3 = −13.
Since the result is a negative, we write negative on the cell that correspond the factor 2𝑥 − 3 and
intervals (−∞, −2). The same process goes to the other cells.

For the last row, since it is a quotient of the first two factors above then all you have to do is divide
the sign found on the cells above it.
Example:
Under the intervals (−∞, −2), the signs are both negative. We know that, the quotient of both
negative numbers is positive.

Step 5: State the solution set


Solution Explanation
𝟑 Note that we are looking for the values of 𝑥 where
(−∞, −𝟐) ∪ [ , ∞) 2𝑥−3
𝟐 ≥ 0.
𝑥+2

𝑥 4 2 − 8𝑥 As seen from the table, the rational inequality has


+ = values greater than 0 (positive) on the intervals
𝑥+2 𝑥−2 4 − 𝑥2
3
(−∞, −2) and ( , ∞).
2
Since, 𝑥 = −2 is a restricted value for the
denominator, so it is not part of the solution set
3
but 𝑥 = .
2

53 | G e n e r a l M a t h e m a t i c s
Guided Practice
Direction: For rational equation, find the unknown variable that makes the equation true. Then for
rational inequality, find the solution set. Show complete solutions.
3 1
1. +𝑥 = 4
𝑥+1

3 1
Given: +𝑥≤4
𝑥+1
Step 1:

Step 2:

54 | G e n e r a l M a t h e m a t i c s
Step 3:

Step 4:

Step 5:

55 | G e n e r a l M a t h e m a t i c s
Reflection
Questions:
1. How did you find the activity?
_______________________________________________________________________________
_______________________________________________________________________________
___________________________________________________________________________.

2. What challenges have you encountered while doing the activity?


_______________________________________________________________________________
_______________________________________________________________________________
___________________________________________________________________________.

3. How did you resolve the challenges you encountered?


_______________________________________________________________________________
_______________________________________________________________________________
___________________________________________________________________________.

4. What were the 3 most important things you have learned from this lesson? Explain each briefly.
a. _____________________________________________________________________.
b. _____________________________________________________________________.
c. _____________________________________________________________________.

56 | G e n e r a l M a t h e m a t i c s
Action
Activity 13: My Story to Tell
Directions: Think of an event that greatly leave an impact in your life, it could be a stumbling block or it
could be a story of success. Then write the ways on how you overcome or achieved it.

57 | G e n e r a l M a t h e m a t i c s
Evaluation
Test I.
Directions: Determine whether the given is a rational function, rational equation, rational inequality, or
none of these. Write your answer on the space provided before each number.

1 1 2
1. + 2𝑥 = 3
𝑥

𝑥+2
Given: ≥4
2𝑥
Step 1:

Step 2:

58 | G e n e r a l M a t h e m a t i c s
Step 3:

Step 4:

Step 5:

59 | G e n e r a l M a t h e m a t i c s
Lesson 6 Graphing Rational Functions
LESSON PRE-ASSESSMENT
Instruction: Determine the graph of the given functions. Write the letter of the function notation of
your choice on the space provided.

𝑥+2
A. 𝑒(𝑥) =
𝑥 2 −𝑥−6

𝑥 2 −2𝑥+1
B. 𝑎(𝑥) =
_____1. 𝑥−3

9𝑥+2
C. 𝑐(𝑥) =
3𝑥+12

_____2.

2𝑥+3
D. 𝑟(𝑥) =
3𝑥+1

_____3.
2𝑥−7
E. 𝑓(𝑥) =
3𝑥+5

_____4.

60 | G e n e r a l M a t h e m a t i c s
Context
Learning Competencies
At the end of the unit, the learners can:
✓ distinguish rational function, rational equation, and rational inequality by:
▪ identifying the characteristics of rational function, rational equation and rational
inequality
▪ giving examples of rational function, rational equation and rational inequality
✓ represent rational function through its: (a) table of values, (b) graph, and (c) equation by:
▪ identifying important details such as x-values that would actually satisfy the function
▪ substituting the values into the x of the function to identify the y-values
✓ find domain and range of rational function by:
▪ finding the restrictions of x values through equating the denominator to 0 and solve for x
(domain)
▪ solving x in terms of y (range)
✓ determine the (a)intercepts, (b) zeroes; and (c) asymptotes of rational functions by:
▪ identifying the importance of zeroes, intercepts and asymptotes to rational function
▪ solving specific unknowns to find the values asked
✓ graph rational function by:
▪ identifying the domain and range of the rational function
▪ identifying the intercepts, zeroes and asymptotes of the function
▪ plotting the points on Cartesian plane
▪ connecting the points to form a graph

Value Integration: Competence

61 | G e n e r a l M a t h e m a t i c s
Experience
Prelection
Directions: In the set of real numbers, determine which value/s of the given variable make/s the
expression on each item undefined. Write your answers on the space provided.

Expressions Values

1. 1
𝑥
2. 2
3𝑦 − 1

3. 3
𝑏 2 − 25
4. 𝑚2 − 3𝑚
2
5. 3
𝑘 2 + 7𝑘 − 18

Concept Notes
Rational Expressions – is an expression that can be written as a ratio of two polynomials.
Meaning, the numerator and denominator are both polynomials.

Examples/Non-Examples Explanation
𝑥2 + 3𝑥 + 2 It is a rational expression because it is a ratio of two
𝑥+4 polynomials.
1 Rational expression because the numerator 1 is a
3𝑥2 polynomial
2 Rational expression because it is a ration of two
𝑥 + 4𝑥 − 3
2 polynomials.
ξ𝑥 + 1 Not a rational expression since the numerator is not a
polynomial
𝑥3 − 1

62 | G e n e r a l M a t h e m a t i c s
Finding the INTERCEPTS, DOMAIN and RANGE of RATIONAL FUNCTION

Given:
2𝑥 + 3
𝑓(𝑥) =
3𝑥 + 1
The zeroes of a function are the values of 𝑥 which make the function zero. The real numbered zeroes
are also x-intercepts of the graph of the function.
Solution Explanation
2𝑥 + 3 Let, 𝑓(𝑥) = 0.
0= Substitute 0 into 𝑓(𝑥) 𝑜𝑟 𝑦.
3𝑥 + 1
0(3𝑥 = 1) = 2𝑥 + 3 Cross multiply, then solve for x.
0 = 2𝑥 + 3 Result after multiplying since, 0(3𝑥 + 1) = 0.
Transpose 3 to the left side then change the sign to
−3 = 2𝑥 negative.
(Concept of Addition Property of Equality)
−3 2𝑥 Divide both sides by 2.
=
2 2
−3 2
=𝑥 Since, = 1.
2
2
3
(− , 0)
2 𝒙 − 𝒊𝒏𝒕𝒆𝒓𝒄𝒆𝒑𝒕.

The 𝒚 − 𝒊𝒏𝒕𝒆𝒓𝒄𝒆𝒑𝒕 is the function value when 𝑥 = 0.


Solution Explanation
2(0) + 3
𝑓(𝑥) = 𝑦 = Let, 𝑥 = 0.
3(0) + 1
𝑦=3 Simplifying

(0, 3) 𝒚 − 𝒊𝒏𝒕𝒆𝒓𝒄𝒆𝒑𝒕
The domain of the function is the set of all values that the variable 𝑥 can take or all permissible values
of x.
Solution Explanation
2𝑥 + 3
𝑓(𝑥) = Given
3𝑥 + 1
3𝑥 + 1 = 0 Take the denominator and equate to 0.
3𝑥 = −1 Transpose 1 to the right side and change the sign.
Divide both sides by 3.
1 1
𝑥=− 𝑥 = − 3 is the VERTICAL ASYMPTOTE of the function (restriction
3
of x-values).
1
{𝑥/𝑥 ≠ − , 𝑥 ∈ ℝ}
3
Domain
It is read as “The set of x such that x is not
1
equal to − 3 wherein x is an element of real
numbers.

63 | G e n e r a l M a t h e m a t i c s
The range of a function is the set of all values that 𝑓(𝑥) can take.
Solution Explanation
2𝑥 + 3 Given
𝑓(𝑥) =
3𝑥 + 1
2𝑥 + 3 Change 𝑓(𝑥) 𝑡𝑜 𝑦
𝑦=
3𝑥 + 1
2𝑥 + 3 Solve 𝑥 in terms of 𝑦
𝑦=
3𝑥 + 1
𝑦(3𝑥 + 1) = 2𝑥 + 3 Cross multiply
3𝑥𝑦 + 𝑦 = 2𝑥 + 3 Distributive property
3𝑥𝑦 − 2𝑥 = −𝑦 + 3 Transpose 2x to the left side and y to the right side, and
change their sign. The goal is to combine terms with x
variable.
𝑥(3𝑦 − 2) = −𝑦 + 3 Factor out the variable x.

𝑥(3𝑦 − 2) −𝑦 + 3
=
3𝑦 − 2 3𝑦 − 2 Divide both sides by 3𝑦 − 2.

−𝑦 + 3
𝑥= Simplify
3𝑦 − 2
−𝑥 + 3 Interchange 𝑥 and 𝑦.
𝑦=
3𝑥 − 2
3𝑥 − 2 = 0 Take the denominator and equate to 0.
3𝑥 = 2 Transpose −2 and change its sign.
2 2
𝑦 = is the HORIZONTAL ASYMPTOTE of the function
𝑥= 3
3 (restriction for the range).
2
{𝑦/𝑦 ≠ , 𝑦 ∈ ℝ}
3
It is read as “The set of y such that y is not Range
2
equal to wherein y is an element of real
3
numbers.

Let us venture formally the asymptotes:

Asymptotes
The line partition on the graph is known as asymptotes.
It is a line that being approached to by the graph of the function.

Vertical Asymptotes
• If the rational function has restrictions on its domain, then it will have vertical asymptote.
• It is a vertical line (sometimes represented by a dash line) which serves as the boundary of
the graph.

64 | G e n e r a l M a t h e m a t i c s
Horizontal Asymptote
• If the rational function has restrictions on its range, then it will have a horizontal asymptote.
• It is a horizontal line (sometimes represented by a dash line) which serves as the boundary
of the graph.

Rules on Finding the Horizontal Asymptote:


𝒑(𝒙)
Consider 𝒇(𝒙) = , 𝒒(𝒙) ≠ 𝟎.
𝒒(𝒙)
Rule 1: If the leading coefficient 𝒂 and 𝒃 of 𝒑(𝒙) and 𝒒(𝒙) respectively, have the same degree
𝒂
then the horizontal asymptote is 𝒚 = .
𝒃
Example Solution/Explanation
𝟐𝒙 + 𝟏 Leading terms are terms with the highest
𝒚=
𝒙−𝟑 exponent/degree. In this case, we have 2𝑥 for
the numerator and 𝑥 for the denominator
since the have the same exponent of 1.
We divide their numerical coefficient which
are 2 and 1. So,
𝟐
𝒚 = = 𝟐.
𝟏
Therefore,
Horizontal Asymptote: 𝒚 = 𝟐.

Rule 2: When the degree of 𝒑(𝒙) is lesser than the degree of 𝒒(𝒙), the horizontal asymptote is at
𝒚 = 𝟎 or the x-axis.
Example Solution/Explanation
𝟐 See that the degree of the numerator is lesser
𝒙 +𝟏
𝒚= 𝟒
𝒙 −𝟓 than of the denominator, 2 < 4.
Thus,
Horizontal Asymptote: 𝒚 = 𝟎.
Rule 3: When the degree of 𝒑(𝒙) is greater than the degree of 𝒒(𝒙), then the horizontal asymptote
does not exist.
Example Solution/Explanation
𝟓 Note that the degree (highest exponent) of
𝒙 +𝟕
𝒚= the numerator is greater than of the
𝒙+𝟐
denominator, 5 > 1.
Then obviously, the function has no horizontal
asymptote.

There is a special case wherein if the degree of


the numerator is 1 greater than of the
denominator, then oblique asymptote can be
found by dividing 𝑝(𝑥) and 𝑞(𝑥).

65 | G e n e r a l M a t h e m a t i c s
Graphing of Rational Function
When graphing, all you have to do is to plot the necessary points and be aware of the restrictions.

66 | G e n e r a l M a t h e m a t i c s
Guided Practice
Direction: For the function, find intercepts and asymptotes. State the domain and range then sketch
the graph. Show complete solution. Domain, Range, 𝑥-intercept, and 𝑦-intercept are equivalent to 2
points each. Vertical and Horizontal Asymptotes are equivalent to 1 point each. The sketch of the graph
is equivalent to 5 points.
𝒙+𝟐
𝒈(𝒙) =
𝒙𝟐 − 𝒙 − 𝟔

Domain: Range:

𝑥 − 𝑖𝑛𝑡𝑒𝑟𝑐𝑒𝑝𝑡: 𝑦 − 𝑖𝑛𝑡𝑒𝑟𝑐𝑒𝑝𝑡:

Vertical Asymptote: Horizontal Asymptote:

Sketch of the Graph:

67 | G e n e r a l M a t h e m a t i c s
Reflection
Questions:
1. How did you find the activity?
_______________________________________________________________________________
_______________________________________________________________________________
___________________________________________________________________________.

2. What are the challenges you encounter while doing the activity? Why do you say so?
_______________________________________________________________________________
_______________________________________________________________________________
___________________________________________________________________________.

3. How did you resolve the challenges you encountered?


_______________________________________________________________________________
_______________________________________________________________________________
___________________________________________________________________________.

68 | G e n e r a l M a t h e m a t i c s
Action
Activity 14: Story highlights!
Directions: Take a look around you. Take a picture of anything you can relate to graphing of rational
functions (it could be the process or the concept, i.e. graph). Print it, then paste it on the space provided
below. You may also draw it. Write a good caption for your picture. Enjoy wandering!

Criteria for Grading:


Originality 3 points
Relation to the Topic 4 points
Quality of Image 2 points
Caption 1 point
TOTAL 10 points

69 | G e n e r a l M a t h e m a t i c s
Evaluation
Directions: For the function, find intercepts and asymptotes. State the domain and range then sketch
the graph. Show complete solution. Domain, Range, 𝑥-intercept, and 𝑦-intercept are equivalent to 2
points each. Vertical and Horizontal Asymptotes are equivalent to 1 point each. The sketch of the graph
is equivalent to 5 points.
𝒙+𝟐
𝒈(𝒙) =
𝒙𝟐 − 𝒙 − 𝟔

Domain: Range:

𝑥 − 𝑖𝑛𝑡𝑒𝑟𝑐𝑒𝑝𝑡: 𝑦 − 𝑖𝑛𝑡𝑒𝑟𝑐𝑒𝑝𝑡:

Vertical Asymptote: Horizontal Asymptote:

Sketch of the Graph:

70 | G e n e r a l M a t h e m a t i c s
Lesson 7 One-to-One and Inverse Functions
LESSON PRE-ASSESSMENT
Directions: Read and understand each item carefully. Encircle the letter of the correct answer.

1. The following are one-to-one functions, EXCEPT


a. b.

c. d.

2. A line test that can determine if the given graph is one-to-one.


a. Horizontal asymptote, the line 𝑦 = 𝑏.
b. Vertical asymptote, the line 𝑥 = 𝑎.
c. Horizontal line test
d. Vertical line test3.
1 2
3. Given the expressions , 5𝑥, 𝑎𝑛𝑑 . Find the Least Common Denominator (LCD).
2𝑥 𝑥
e. 2𝑥
f. 2𝑥 2
g. 5𝑥
h. 𝑥
4. Identify which of the statements is/are true.
i. One-to-one functions have inverses.
ii. Inverse functions are one-to-one

a. i
b. ii
c. i and ii
d. none of the above

71 | G e n e r a l M a t h e m a t i c s
Context
Learning Competencies
At the end of the unit, the students can:
✓ represent real-life situations using one-to-one functions by:
▪ identifying the characteristics of one to one function.
▪ giving examples of one to one function.
✓ determine the inverse of one-to-one function by:
▪ changing 𝑓(𝑥) to 𝑦.
▪ solving 𝑥 in terms of 𝑦, then interchange 𝑥 𝑎𝑛𝑑 𝑦.
✓ represent an inverse function through its (a) table of values, and (b) graph
▪ identifying the permissible values 𝑓(𝑥) through values of 𝑥.
▪ using horizontal line test.
✓ graphs inverse functions
▪ identifying domain and range.
▪ identifying values.
✓ solve problems using inverse functions.
▪ relating the problems to the concepts of one-to-one and inverse function.

Value Integration: Character

Experience
Prelection
Activity 15: Mind Web!
Directions: You are a Robotech! You are known for your honest and factual works. Now, your task is to
process the image shown by writing your observations about it. Enjoy!

72 | G e n e r a l M a t h e m a t i c s
Concept Notes
FUN FACTS!
▪ Finding the inverse of a function means undoing each operation in the function.
▪ Only one-to-one function has a unique inverse.

Definition: ONE-TO-ONE FUNCTION

A function is one-to-one if and only if every element in its range corresponds to one and only one
element in its domain.

Representations:

1. The relation pairing an SSS member to his or her SSS number.

Explanation: Each SSS member assigned a unique SSS number. Thus, the relation is a function.
Further, two different members cannot be assigned the same SSS number. Thus the function is one-
to-one.

2. The relation pairing a real number to its square.

Explanation: Each real number has a unique perfect square. Thus, the relation is a fnction. However,
two different real numbers such as 2 and -2 may have the same square. Thus, the function is not one-
to-one.

3. The relation pairing an airport to its airport code. Airport codes are three letter codes used to
uniquely identify airports around the world and prominnently displayed on check-in bags to
denote the destination of these bags. Here are some examples of airport codes:
MNL-Ninoy Aquino International Airport (All terminals)
CEB –Mactan-Cebu International Airport
DVO – Francisco Bangoy International Airport (Davao)

73 | G e n e r a l M a t h e m a t i c s
GRAPHS OF ONE-TO-ONE FUNCTION

Definition:

A function is one-to-one if each horizontal line test does not intersect the graph at more than one
point.

A function has an inverse if it is one-to-one. But how can we check if a function is one-to-one?
Consider some examples and non-examples ofone-to-one function below.

Examples Non-examples

𝑓(𝑥) = ξ𝑥 𝑓(𝑥) = 𝑥 2

1 𝑓(𝑥) = |2𝑥 |
𝑓(𝑥) =
𝑥
𝑓(𝑥) = 2𝑥 𝑓(𝑥) = 2 + ⌊𝑥 ⌋

𝑓(𝑥) = 3𝑥 + 5 𝑓(𝑥) = 𝑥 4 − 1

𝑓(𝑥) = 𝑥 3 − 1 𝑓(𝑥) = 2𝑠𝑖𝑛𝑥

Explanation:

A function 𝑎(𝑥) is an inverse of a function 𝑏(𝑥) (vice versa) if and only if the elements in the domain
of 𝑎(𝑥) are the elements in the range of 𝑏(𝑥) and the elements in the range of 𝑎(𝑥) are the elements
in the domain of 𝑏(𝑥).

74 | G e n e r a l M a t h e m a t i c s
Illustration:

𝑎(𝑥) 𝑏(𝑥)

1 a

2 b

3 c

Note that:
The domain of 𝑎(𝑥): {1,2,3} The domain of 𝑏(𝑥): {𝑎, 𝑏, 𝑐}
The range of 𝑎(𝑥): {𝑎, 𝑏, 𝑐} The range of 𝑏(𝑥): {1,2,3}

75 | G e n e r a l M a t h e m a t i c s
EXAMPLE 1: Find the inverse of 𝑓(𝑥) = 3𝑥 + 1
Solution Explanation
𝑦 = 3𝑥 + 1 First step. Change 𝑓(𝑥) into 𝑦.
𝑥 = 3𝑦 + 1 Second step. Interchange 𝑥 and 𝑦.
𝑥 = 3𝑦 + 1 Third step. Solve for 𝑦 in terms of 𝑥.
𝑥 − 1 = 3𝑦
𝑥−1
=𝑦
3
𝑥−1 Therefore, the inverse of 𝑓(𝑥) = 3𝑥 + 1 is
= 𝑓−1 (𝑥) 𝑥−1 −1
3 = 𝑓 (𝑥).
3
Example 2: Find the inverse of 𝑓(𝑥) = 𝑥3 − 2.
Solution Explanation

𝑦 = 𝑥3 − 2 First step. Change 𝑓(𝑥) into 𝑦.

𝑥 = 𝑦3 − 2 Second step. Interchange 𝑥 and 𝑦.

𝑥 + 2 = 𝑦3 Third step. Solve for 𝑦 in terms of 𝑥.


3
ξ𝑥 + 2 = 𝑦
−1 3
𝑓 (𝑥) = ξ𝑥 + 2 Therefore, the inverse of 𝑓(𝑥) = 𝑥 3 − 2 is
𝑓−1 (𝑥) = ξ𝑥 + 2.
3

Given the graph of a one-to-one function, the graph of its inverse can be obtained
by reflecting the graph about the line 𝑦 = 𝑥.
Example 1:
𝑓(𝑥) = 2𝑥 + 1 The given function
𝑥−1
𝑓−1 (𝑥) = The inverse of the function
3

Domain:
The
domian
of the
function
is the set
of real
numbers.

Range: The range of the function is the set of real numbers

76 | G e n e r a l M a t h e m a t i c s
Example 2: Find the Inverse Function
𝑓(𝑥) = 𝑥 3 − 2

A. Determine whether the function is one-to-one

Solution Explanation
)
Since 𝑓(𝑥1 = 𝑓(𝑥2 ) implies 𝑥1 = 𝑥2 Meaning, in every value of 𝑥, there is a unque
then 𝑓(𝑥) is one to one function. output of 𝑓(𝑥).

B. If the Function is one-to-one, find its inverse.

Solution Explanation
𝑦 = 𝑥3 − 2 First step. Change 𝑓(𝑥) into 𝑦.

𝑥 = 𝑦3 − 2 Second step. Interchange 𝑥 and 𝑦.

𝑥 + 2 = 𝑦3 Third step. Solve for 𝑦 in terms of 𝑥.


3
ξ𝑥 + 2 = 𝑦
𝑓−1 (𝑥) = ξ𝑥 + 2
3
Therefore, the inverse of 𝑓(𝑥) = 𝑥 3 − 2 is
𝑓−1 (𝑥) = ξ𝑥 + 2.
3

C. Find the domain and range of the inverse function

Solution Explanation
Domain: All real numbers The given function is a polynomial function.

Range: All real number The given function is a polynomial function.

A. Sketch the graph

Solution Explanation
You can use geogebra application in graphing
the function.

77 | G e n e r a l M a t h e m a t i c s
Application of Inverse Function

Problem: Converting Temperature


When converting temperature readings in Fahrenheit to degree Celsius, we use the formula,
5
𝐶° = (𝐹° − 32).
9

a. Does the equation represent one-to-one function?

Solution:
The equation can be written into the form 𝑦 = 𝑚𝑥 + 𝑏. Since it is a linear then it is a one-
to-one function.
5 160
𝐶° = 𝐹 ° −
9 9
b. Write the inverse of the function

Solution:
5
𝐶° = (𝐹° − 32) Multiply both sides by 9/5.
9
9
𝐶° = 𝐹° − 32 Transpose −32 and change the sign.
5
9
𝐶° + 32 = 𝐹° inverse of the Function
5

c. Convert the following temperature to Fahrenheit.

c.1. 30° c.2. 18°

Solution: Solution:
9 9
(30 + 32) = 𝐹° Substitute 30 into C. (18 + 32) = 𝐹° Substitute
5 5
18 into C.
111.6 = 𝐹 ° 90 = 𝐹 °
111.6 𝐹 ° 90 𝐹 °

78 | G e n e r a l M a t h e m a t i c s
Guided Practice
Directions: Given the function of 𝑓(𝑥) at the center of the organizer, provide your answer by writing it
on the box that corresponds to the question/statement below.

Inverse of 𝑓(𝑥) 4 pts 𝑓(𝑥) 2 pts Inverse of 𝑓−1 (𝑥) 4 pts


Domain: ___________

Range: _____________

𝑓−1 (𝑥):
Is 𝑓 (𝑥) a one-to-one function or 𝟐 Is 𝑓 −1 (𝑥) a one-to-one function
not? 1 pt 𝒇(𝒙) = or not? 1 pt
𝟑𝒙 − 𝟔

Graph of 𝑓(𝑥) 3 pts Graph of 𝑓−1 (𝑥) 3 pts

79 | G e n e r a l M a t h e m a t i c s
Reflection
Questions:
1. How did you fill the table with your own knowledge?
________________________________________________________________________________
________________________________________________________________________________
________________________________________________________________________________
________________________________________________________________________________
__________________________________________________________________.

2. What were the challenges you encountered when doing the tasks?
________________________________________________________________________________
________________________________________________________________________________
________________________________________________________________________________
________________________________________________________________________________
__________________________________________________________________.
3. Were you able to resolve the challenges encountered? If no, why? If yes, how did you do it?
________________________________________________________________________________
________________________________________________________________________________
________________________________________________________________________________
________________________________________________________________________________
__________________________________________________________________.

80 | G e n e r a l M a t h e m a t i c s
Action
Activity 16: Point of View
Directions: A one-to-one function always has an inverse which means it generates a mirror image of its
inverse. An inverse function may be different in forms (equation) but it has the same properties with
the given function. In a similar manner, our life may be represented by different versions of ourselves
depending on what a situation requires of us. We are always welcome to lookback and be our self again.

Now, your task is to look for a picture of a symmetric object, cut it along the axis of symmetry. Paste
the half image in the Cartesian plane and graph the other half. Write a short explanation about the
chosen object or picture. Name of Object = 2pts, Image = 5pts, Explanation = 3pts

EXAMPLE: BUTTERFLY

At some point in my life I get


tired of what is happening
𝑓(𝑥) = 𝑥 around me. Vision of my
dreams seems to fade slowly
however, the people that I
encounter every day helps
realize that there is more
reasons to be happy. Every day
I can improve, hope, change,
and live an amazing and
colorful life, just like a
butterfly.

81 | G e n e r a l M a t h e m a t i c s
Evaluation
Directions: Answer the following items completely. Write your answer on the space provided after each
item.

𝑥−5
1. Given the function 𝑓(𝑥) = ,
𝑥+2

a. Determine whether the function is one-to-one. (1 point)

b. If the function is one-to-one, find its inverse. (3 points)

c. Find the domain and range of the inverse function (2 points)

d. Sketch the graph (2 points)

82 | G e n e r a l M a t h e m a t i c s
2. A piece-wage worker is given an hourly rate of Php50.00 plus an additional pay of Php1.25 per
unit produced. His hourly wage 𝑤(𝑥) is modeled by the function, 𝑤(𝑥) = 50 + 1.25𝑥, where
𝑥 is the number of units produced per hour.

a. What is the inverse of 𝑤(𝑥) = 50 + 1.25𝑥,? (3 points)

b. What do 𝑥 and 𝑤−1 (𝑥) represent in the inverse function? (2 points)

c. If the worker receives Php75 an hour, how many units of products did he produce? (2 points)

83 | G e n e r a l M a t h e m a t i c s
Lesson 8 Representing Real-Life Situations Using Exponential Functions
LESSON PRE-ASSESSMENT
Directions: The following questions are about representing exponential functions. Read each item
carefully and choose the best answer among the choices. Encircle the letter of your choice.

1. Exponential functions have the form 𝑓(𝑥) = 𝑎 𝑥 . “𝑎” is the __________ of the exponential
expression 𝑎𝑥 .
A. Exponent
B. Asymptote
C. Base
D. Intercept

2. Which of the following represents an exponential function?

A. Population Growth
B. Radioactive Decay
C. Compound Interest
D. All of these

3. In the given table below, what is the relationship of the ordered pairs?
𝒙 9 17 25 33
𝒚 -6 -24 -96 -384
A. Exponential
B. Linear
C. Linear and Exponential
D. Neither Linear nor Exponential

1
1 100
4. A radioactive isotope decays at the rate indicated by exponential function 𝐴(𝑡) = 800 ( ) ,
2
where ′𝑡 ′ is the time in years and 𝐴(𝑡) is the amount of the isotope, in grams, remaining. What
is the initial mass of the isotope?
A. 900 grams
B. 800 grams
C. 1,600 grams
D. 750 grams

5. Benjamin invests ₱10,000 in savings account that earns 5% interest each year. Which
expression gives the balance in the account after 3 years?
A. 10,000 + (1 + 0.05)(1 + 0.05)(1 + 0.05)
B. 10,000 + (1 + 0.05) + (1 + 0.05) + (1 + 0.05)
C. 10,000(0.05)(0.05)(0.05)
D. 10,000(1 + 0.05)(1 + 0.05)(1 + 0.05)

84 | G e n e r a l M a t h e m a t i c s
Context
Learning Competencies
At the end of the lesson, the learner is able to:
✓ transform exponential functions
✓ represent real-life situations using Exponential functions
▪ Exponential growth and decay
▪ Compound interest
▪ The natural exponential function

Value Integration: Compassion

Experience
Prelection
Activity 17: COVID-19 Pandemic

TABLE (CoVid-19 Cases) Area Covered: Philippines .

Month January February March April May June July 14


Number of
Confirmed 2010 1826 58,087 72,942 116,020 163,859 112,258
Cases

85 | G e n e r a l M a t h e m a t i c s
Directions: Observe the data provided above and answer the following questions:

(a) What happens to the number of cases (b) What kind of function do you think the
from January to July? data represents?

____________________________________ ____________________________________
____________________________________ ____________________________________
____________________________________ ____________________________________

Concept Notes

Just like the data on COVID-19


cases, there are other real-life
situations where a rapid change
(increase or decrease) in the data is
observable. When this happens, we
can say that the situation represents
an exponential function.

Definition.
An exponential function with base bis a function of the
form 𝑓(𝑥) = 𝑏 𝑥 or 𝑦 = 𝑏 𝑥 , where 𝑏 > 0, 𝑏 ≠ 1.

Definition
Let 𝑏 be a positive number not equal to 1. A
transformation of an exponential function with base 𝑏
is a function of the form
𝑔(𝑥) = 𝑎 ∙ 𝑏 𝑥−𝑐 + 𝑑
where 𝑎, 𝑐, and 𝑑 are real numbers.

86 | G e n e r a l M a t h e m a t i c s
Example of
1. 256 = 4𝑥−5
Exponential Function 2. 52𝑥 = 253𝑥+2
(Equation)
1 𝑥+2
3. ( ) = 36𝑥+9
3

Many applications involve transformations of exponential functions. Some of the most common
applications in real-life of exponential functions and their transformations are population growth,
exponential decay, and compound interest.

Population Growth

On several instances, scientists will


start with a certain number of bacteria or
animals and watch how the population
grows. For instance, if the population
doubles every 3 days, this can be represented
as an exponential function.

EXAMPLE 1
Let 𝑡 = time in days. At 𝑡 = 0, there were initially 20 bacteria. Suppose that the bacteria double every
100hours. Give an exponential model for the bacteria as a function of 𝑡.

Solution:
Initially,
𝒕 Number of Bacteria
Therefore, an exponential
0 20 20
model for this situation is
100 20(2) 40
𝑡
200 20(2)2 80 𝑦 = 20(2)100
300 20(2)3 160
400 20(2)4 320

Exponential Models and Population Growth.


Suppose a quantity 𝑦 doubles every 𝑇 units of time. If 𝑦0 is the initial amount,
then the quantity 𝑦 after 𝑡 units of time is given by
𝒕
𝒚 = 𝒚𝟎 (𝟐) ൗ𝑻 .

87 | G e n e r a l M a t h e m a t i c s
Exponential Decay
Exponential functions can be used to model radioactive decay

The half-life of a radioactive substance is


Recall that, the time it takes for half of the substance to
decay.

EXAMPLE 2
Suppose that the half-life of a certain radioactive substance is 10 days and there are 10g initially,
determine the amount of substance remaining after 30days.

Solution:

Let 𝑡 = time in days. We use the fact that the mass is halved every 10days (from definition of half-life).
Thus, we have:

Initially,

𝑡 Amount of Substance
Therefore, an exponential model
0 10𝑔 𝑡ൗ
1 10
10 5𝑔 for this situation is 𝑦 = 10 ( ) .
2
20 2.5𝑔
30 1.25𝑔

Compound Interest

A starting amount of money (called the principal) can be invested at a certain interest
rate that is earned at the end of a given period of time (such as one year). If the interest
rate is compounded, the interest earned at the end of the period is added to the
principal, and this new amount will earn interest in the next period. The same process
is repeated for each succeeding period: interest previously earned will also earn
interest in the next period.

EXAMPLE 3
Mrs De la Cruz invested ₱100,000 in a company that offers 6% interest compounded annually. How
much will this investment be worth at the end of each year for the next five years?

88 | G e n e r a l M a t h e m a t i c s
Solution
Let 𝑡 be the time in years. Then we have
Initially,

𝒕 Investments
0 ₱100,000
1 ₱100,000(1.06) = ₱106,000 Therefore, an exponential
model this situation is
2 ₱106,000(1.06) = ₱112,360
𝑦 = 100,000(1.06)𝑡
3 ₱112,360(1.06) ≈ ₱119,101.60
4 ₱119,101.60(1.06) ≈ ₱126,247.70
5 ₱126,247.70 (1.06) ≈ ₱133,822.56

Compound Interest.
If principal 𝑃 is invested at an annual rate of 𝑟, compounded annually, then the amount after 𝑡
years is given by
𝐴 = 𝑃(1 + 𝑟)𝑡 .

EXAMPLE 4
Based on example #3, is it possible for Mrs. De la Cruz to double her money in 8 years? in 10 years?

Solution:
Using the model 𝑦 = 100,000(1.06)𝑡 , substitute 𝑡 = 8 and 𝑡 = 10.

If 𝑡 = 8, If 𝑡 = 10,
then 𝑦 = ₱100,000(1.06)8 then 𝑦 = ₱100,000(1.06)10
𝑦 ≈ ₱159,384.81 𝑦 ≈ ₱179,084.77

Since her money still has not reached ₱200,000 after 10years,

therefore, she has not doubled her money during this time.

The Natural Exponential Function

Definition

The natural exponential function is the function 𝑓(𝑥) = 𝑒 𝑥

89 | G e n e r a l M a t h e m a t i c s
EXAMPLE 5
A large slab of meat is taken from refrigerator and placed in a pre-heated oven. The temperature 𝑇 of
the slab 𝑡 minutes after being placed in the oven is give by 𝑇 = 170165𝑒 0.006𝑡 degrees Celsius.
Construct a table of values for the following values of 𝑡: 0, 10, 20, 30, 40, 50, 60, and interpret your
results. Round off values to the nearest integer.

Solution
Table of Values:
𝑡 0 10 20 30 40 50 60

𝑇 5 15 24 32 40 47 54

Therefore, the slab of the meat is increasing in temperature at roughly the same rate.

90 | G e n e r a l M a t h e m a t i c s
Guided Practice
Directions: Exponential functions can be applied in growth, decay, and investment problems. Below
are some situations representing exponential functions. For each item, give an exponential model for
the situation and answer the question that follow. Each item is equivalent to 2 points.

91 | G e n e r a l M a t h e m a t i c s
Reflection
Questions:
1. On a scale of 1 (easiest) to 10 (hardest), how will you rate the level of difficulty of the activity?
Why do you say so?
_______________________________________________________________________________
_______________________________________________________________________________
2. Which part of the activity do you like the most? least? Why?
_______________________________________________________________________________
_______________________________________________________________________________
3. What part of the activity do you find confusing?
_______________________________________________________________________________
_______________________________________________________________________________

Action
Activity 18

92 | G e n e r a l M a t h e m a t i c s
SLOGAN/ COLLAGE SCORING RUBRIC
CRITERIA 𝟏 𝟐 𝟑 SCORE

Content is incorrect or Content is somewhat Content is excellent


Content inappropriate for the relating to the and appropriate for the
tribute/topic. tribute/topic. tribute/topic.
appropriate or
Slogan / Catchy and
Lacks originality acceptable in quality
Collage memorable.
or quantity
Effective
Promotion / Does not promote Do not feel strongly ad/commercial; makes
Effectiveness tribute ; ineffective either way audience want to
sponsor the tribute.
Purpose is
Purpose not entirely
Purpose confusing/unclear or Purpose is clear
clear
lacking
Little or no visual aides,
or visual is Some use of visual Excellent use of visual
Use of Visual
inappropriate or aides aides
ineffective

Creativity Little or no creativity Somewhat creative Creative and Original.

TOTAL SCORE .

OUTPUT HERE! (Do not go beyond the box provided for your output.)

93 | G e n e r a l M a t h e m a t i c s
Evaluation
Directions: Given the following real-life situations representing exponential functions, give a
mathematical model that best represents the situation and answer the questions that follow.
(5pts each) (a) 2pts and (b) 3pts

1. Bacteria can multiply at an alarming rate when each bacteria split into two new cells, thus doubling. We start
with only one bacterium which can double every hour.
a. Give the exponential model for the situation.
b. How many bacteria will we have after 10 years?

2. ₱10,000is invested at 2% compounded annually.


a. Give the exponential model for the situation.
b. What is the amount after 12 years.

94 | G e n e r a l M a t h e m a t i c s
Lesson 9 Solving Exponential Functions, Equations, and Inequalities
LESSON PRE-ASSESSMENT
Directions: Determine whether the given is an exponential function, an exponential equation, an
exponential inequality or none of these.

a) 𝑓(𝑥) = 2𝑥 3 _______________________________

b) 𝑓(𝑥) = 2𝑥 _______________________________

c) 𝑦 = 𝑒 𝑥 _______________________________

d) 22 (5𝑥+1 ) = 500 _______________________________

e) 625 ≥ 5𝑥+8 _______________________________

Context
Learning Competencies:
At the end of the unit, the learner is able to:
✓ distinguish among exponential functions, exponential equations and exponential inequalities.
✓ solve exponential equations and inequalities.

Value Integration: Competence

95 | G e n e r a l M a t h e m a t i c s
Experience
Prelection
Activity 18
Directions: Look into the solutions of John and Peter to the given inequality. Determine who has solved
the problem correctly and which line has had error/s. Write your responses on the space provided.

(1) John and Peter are solving (0.6)𝑥−3 > (0.36)−𝑥−1 . Did anyone get the correct solution? If not, spot
the error or errors.
JOHN PETER

(𝟎. 𝟔)𝒙−𝟑 > (𝟎. 𝟑𝟔)−𝒙−𝟏 (0.6)𝑥−3 > (0.36)−𝑥−1


(𝟎. 𝟔)𝒙−𝟑 > (𝟎. 𝟔𝟐 )−𝒙−𝟏 (0.6)𝑥−3 > (0.62 )−𝑥−1
(𝟎. 𝟔)𝒙−𝟑 > (𝟎. 𝟔)𝟐(−𝒙−𝟏) (0.6)𝑥−3 > (0.6)2(−𝑥−1)

(𝟎. 𝟔)𝒙−𝟑 > (𝟎. 𝟔)−𝟐𝒙−𝟐 (0.6)𝑥−3 > (0.6)−2𝑥−2


𝒙 − 𝟑 > −𝟐𝒙 − 𝟐 𝑥 − 3 > −2𝑥 − 2
𝟑𝒙 > 𝟏 3𝑥 < 1
𝟏 1
𝒙> 𝑥<
𝟑 3

ANSWER:

Recall the following definitions and theorems.

96 | G e n e r a l M a t h e m a t i c s
Concept Notes

Definition.
An exponential expression is an expression of the form 𝑎 ∙
𝑏 𝑥−𝑐 + 𝑑, where 𝑏 > 0, 𝑏 ≠ 1.

Exponential Equation Exponential Inequality Exponential Function

Function of the
An equation involving An inequality involving
Definition form 𝑓(𝑥) = 𝑏 𝑥 , where
exponential expressions. exponential expressions.
𝑏 > 0, 𝑏 ≠ 1.

2 1 𝑓(𝑥) = (1.8)𝑥 or
Example 72𝑥−𝑥 = 52𝑥 − 5𝑥+1 ≤ 0
343 𝑦 = (1.8)𝑥

An exponential equation or inequality can be solved for all 𝑥 values that satisfy the equation or
inequality. An exponential function is not ‘solved’. Rather, it expresses a relationship between
two variables (such as 𝑥 and 𝑦), and can be represented by table of values or a graph.

Some exponential equations can be solved by using that fact that exponential functions are one to one.

One-to-one Property of Exponential Functions


If 𝑥1 ≠ 𝑥2 , then 𝑏 𝑥1 ≠ 𝑏 𝑥2 . Conversely, if 𝑏 𝑥1 = 𝑏 𝑥2 then 𝑥1 = 𝑥2 .

EXAMPLE 1
Solve the equation 4𝑥−1 = 16

Solution

𝟒𝒙−𝟏 = 𝟏𝟔 Given

𝟒𝒙−𝟏 = 𝟒𝟐 Write both sides of the same base.

𝒙−𝟏=𝟐 Solve the exponents

𝒙−𝟏+𝟏=𝟐+𝟏 Solve for x by adding both side by 1

𝒙=𝟑 Simplify

97 | G e n e r a l M a t h e m a t i c s
Checking

You can verify that 𝑥 = 3 is a solution by substituting it back to the original equation.

4𝑥−1 = 16
43−1 = 16
42 = 16
16 = 16

Exponential inequalities can be solved using the following properties.

Property of Exponential Inequalities

If 𝑏 > 1, then the exponential function 𝑦 = 𝑏 𝑥 is increasing for all 𝑥. This means that 𝑏 𝑥 < 𝑏 𝑦
if and only if 𝑥 < 𝑦.

If 0 < 𝑏 < 1, then the exponential function 𝑦 = 𝑏 𝑥 is decreasing for all 𝑥. This means that
𝑏 𝑥 > 𝑏 𝑦 if and only if 𝑥 < 𝑦.

EXAMPLE 2
Solve the inequality 3𝑥 < 9𝑥−2

Solution

𝟑𝒙 < 𝟗𝒙−𝟐 Given

𝟑𝒙 < 𝟑𝟐(𝒙−𝟐) Write both side of the same base

𝟑𝒙 < 𝟑𝟐𝒙−𝟒 Simplify the exponents

𝒙 < 𝟐𝒙 − 𝟒 Solve the exponents

𝒙 − 𝒙 + 𝟒 < 𝟐𝒙 − 𝒙 − 𝟒 + 𝟒 Solve for x

𝟒<𝒙 Simplify

Thus, the solution set is (4, +∝].


(You can verify that 𝑥 = 5 and 6 are solutions, but 𝑥 = 4 and 3 are not.)

98 | G e n e r a l M a t h e m a t i c s
Guided Practice

1) The following items involve exponential expressions. Classify each given as exponential equation or
exponential inequality. Rewrite the given inside the box that corresponds to its category. Each item is
equivalent to 1 point.

2) Solve for 𝑥 for the following exponential expressions. Write your answers on the box provided for
each number. Each item is equivalent to 2 points.

99 | G e n e r a l M a t h e m a t i c s
Reflection
Questions:
1. What was the easiest thing that you did in this lesson? Why?
________________________________________________________________________________
________________________________________________________________________________

2. What was the hardest thing you did and why?


________________________________________________________________________________
________________________________________________________________________________

3. What did you learn, that you think you will use/need in the future?
________________________________________________________________________________
________________________________________________________________________________

Action
Activity 19
Directions: In the past months, the positive cases of CoViD-19 in the world have been rapidly increasing
– representing an exponential function. Below is the summary of data from DOH: [doh.gov.ph] on
CoViD-19 cases in the Philippines in each month. Come up with an equation that represent the data
and predict the total number of confirmed cases for the month of October. How did you come up with
the answer? Equation is equivalent to 3 points while Prediction is equivalent to 2 points.
TABLE (CoVid-19 Cases) Area Covered : Philippines .

Month January February March April May June July 14

Number of
Confirmed 2010 1826 58,087 72,942 116,020 163,859 112,258
Cases

Equation: Prediction:
➢ Given the date above, can you predict the total number of confirmed cases for the month of
October? (You have two options of possible answer)
➢ How did you come up with that answer?

100 | G e n e r a l M a t h e m a t i c s
Evaluation
Direction: Solve for 𝑥 for the following exponential expressions. Write your answers on the box
provided for each number. Each item is equivalent to 5 points.

101 | G e n e r a l M a t h e m a t i c s
Lesson 10 Graphing Exponential Function
LESSON PRE-ASSESSMENT
Direction: The following questions are about exponential functions and their graphs. Match exponential
functions in column A with the correct graph in column B. Write only the letter of your answer on the
blank provided before each number.

COLUMN A COLUMN B

1 𝑥
__________1. 𝑓(𝑥) = 5 ( )
2
a.

__________2. 𝑓(𝑥) = 2𝑥

b.

1 𝑥
__________3. 𝑓(𝑥) = ( )
2
c.

__________4. 2𝑥−1 + 3
d.

__________5. 𝑔(𝑥) = −2𝑥−3

e.

102 | G e n e r a l M a t h e m a t i c s
Context
Learning Competencies:
At the end of the unit, the learner is able to:
✓ represent an exponential function through its table of values, graph and equation
✓ find the domain, range, intercepts, zeroes, and asymptotes of an exponential function
✓ graph exponential functions
▪ graphs of 𝑓(𝑥) = 𝑏 𝑥 if 𝑏 > 1
▪ graphs of 𝑓(𝑥) = 𝑏 𝑥 if 0 < 𝑏 < 1

Value Integration: Competence

Experience
Prelection
Activity 20: CoViD-19 Tracker
Directions: Use the table below as your data for this activity. Plot the points on a Cartesian plane. Let
the students sketch the graph. Let 𝑥 be the month and𝑦 be the Number of confirmed cases. Observed
the graph and answer the question that follows.

TABLE (CoVid-19 Cases) Area Covered : Philippines .

Month January February March April May June July 14

Number of
Confirmed 2010 1826 58,087 72,942 116,020 163,859 112,258
Cases

103 | G e n e r a l M a t h e m a t i c s
QUESTIONS:

(1) What have you observed in the graph?

___________________________________________________________________________
___________________________________________________________________________
___________________________________________________________________.

(2) What kind of function does the graph show?

___________________________________________________________________________
___________________________________________________________________________
___________________________________________________________________.

Concept Notes
We first graph by plotting a few points. Given a function 𝑓(𝑥) = 2𝑥 .

STEP ILLUSTRATION

1 – Create table of 𝑥 −3 −2 −1 0 1 2 3 4
values for 𝑓(𝑥)
𝑓(𝑥) 1 1 1 1 2 4 8 16
8 4 2

2 – (a) Plot the points (a) Plotting of points


found in the table and for 𝑓(𝑥) = 2𝑥 .
(b) connect the points
using a smooth curve.

104 | G e n e r a l M a t h e m a t i c s
(b) Graph of
𝑓(𝑥) = 2𝑥

It can be observed that the function is defined for all values of 𝑥, is strictly increasing, and attains
only positive 𝑦- values. As 𝑥 decreases without bound, the function approaches 0. That is, the line
𝑦 = 0 is a horizontal asymptote.

EXAMPLE 1
1 𝑥
Sketch the graph of 𝑔(𝑥) = ( )
2

Solution
1. 𝑥 −3 −2 −1 0 1 2 3 4
1 1 1 1
𝑓(𝑥) 8 4 2 1
2 4 8 16

2.

1 2
(a) Plotting of points for 𝑓(𝑥) = ( ) (b) Graph of 𝑓(𝑥) = 2𝑥
2

It can be observed that the function is defined for all values of 𝑥, is strictly decreasing, and attains
only positive 𝑦- values. As 𝑥 decreases without bound, the function approaches 0. That is, the line
𝑦 = 0 is a horizontal asymptote.

105 | G e n e r a l M a t h e m a t i c s
In general, depending on whether 𝑏 > 1 or 0 < 𝑏 < 1, the graph of 𝑓(𝑥) = 𝑏 𝑥 has the following
behavior

(a) 𝑏 > 1 (b) 0 < 𝑏 < 1

PROPERTIES OF EXPONENTIAL FUNCTIONS

The following properties of 𝑓(𝑥) = 𝑏 𝑥 , where 𝑏 > 0 and 𝑏 ≠ 1, can be observed from the graph:

(a) The domain is the set ℝ.

(b) The range is the set (0, +∞).


(c) It is a one-to-one function. It satisfies the Horizontal Line Test.

(d) The 𝑦-intercept is 1. There is no 𝑥-intercept.

(e) The horizontal asymptote is the line 𝑦 = 0 (or the 𝑥-axis). There is
no vertical asymptote.

Guided Practice

(a) Complete the following table of values for 𝑓(𝑥) = 2𝑥 and 𝑔(𝑥) = 3𝑥 . Each equation is equivalent
to 2 points.
Solution:
𝒙 −𝟒 −𝟑 −𝟐 −𝟏 𝟎 𝟏 𝟐 𝟑

𝒇(𝒙)

𝒈(𝒙)

106 | G e n e r a l M a t h e m a t i c s
(b) Construct a coordinate plane showing 𝑥 from (−4) to 4 and 𝑦 from (−1) to 30. Sketch the graphs
of 𝑓(𝑥) and 𝑔(𝑥) on this coordinate plane. Compare both graphs. Each graph is equivalent to 2
points.

Solution:

(c) For each function, identify the domain, range, y-intercept, and horizontal asymptote. Each item is equivalent
to 1 point.

Solution:

107 | G e n e r a l M a t h e m a t i c s
Reflection
Questions:
1. What was the easiest thing that you did in this lesson? Why?
_______________________________________________________________________________
_______________________________________________________________________________
__________________________________________________________________.

2. In what way can the lesson help you in your everyday life?
_______________________________________________________________________________
_______________________________________________________________________________
__________________________________________________________________.

3. What skills do you think you were able to enhance in the activity?
_______________________________________________________________________________
_______________________________________________________________________________
___________________________________________________________________.

108 | G e n e r a l M a t h e m a t i c s
Action
Activity 21
Direction: In the past months, the positive cases of COVID-19 in the world have been rapidly increasing
– representing an exponential function. Search for the data of the Number of Positive Cases of COVID-
19 from one of the following countries: China, Japan, Korea, or USA. Plot the data, graph, and answer
the questions that follow. The table is equivalent to 2 points and the graph is equivalent to 2 points.

TABLE (CoVid-19 Cases) Area Covered : _______________________

Month January February March April May June July 14

Number of
Confirmed
Cases

Source: ______________________________________________________________________

Questions:

109 | G e n e r a l M a t h e m a t i c s
Evaluation
Directions: Graphing exponential function requires a step-by-step process. Follow the steps (a), (b), and
(c) to graph the given functions.

1 𝑥 1 𝑥
(a) Complete the following table of values for 𝑓(𝑥) = ( ) and 𝑔(𝑥) = ( ) .
2 3

Solution:

𝒙 −𝟒 −𝟑 −𝟐 −𝟏 𝟎 𝟏 𝟐 𝟑

𝒇(𝒙)

𝒈(𝒙)

(b) Construct a coordinate plane showing 𝑥 from (−4) to 4 and 𝑦 from (−1) to 30. Sketch the graphs
of 𝑓(𝑥) and 𝑔(𝑥) on this coordinate plane. Compare both graphs.

Solution:

(c) For each function, identify the domain, range, 𝑦-intercept, and horizontal asymptote.
Solution:

110 | G e n e r a l M a t h e m a t i c s
Lesson 11 The Logarithmic Function
LESSON PRE-ASSESSMENT
Directions: Read each item and choose the best answer among the choices. Encircle the letter of your
choice. Each item is equivalent to 1 point.

1. What is the base of a common logarithm?

a. 2 b. 8
c. 𝑒 d. 10

2. What is the base of a natural logarithm?

a. 𝑛 b. 𝑒
c. 𝑥 d. 𝑙

3. What is the base of the logarithm log 2 𝑥 = 4?

a. 2 b. 𝑥
c. 4 d. 16

4. Which of the following is NOT a logarithmic function?

a. log 64 𝑥 = 2 b. log 𝑒 𝑥 = 2
c. ln 𝑥 = 4 d. 2𝑥 = 8

5. Which of the following is an application of logarithmic functions?

a. Radioactive Decay b. Acoustics


c. Simple Interest d. Spread of Diseases
.

Context
Learning Competencies
At the end of the lesson, the learners will be able to:
✓ represent real life situation using logarithmic functions; and
✓ solve problems using logarithms.
Value Integration: Character

111 | G e n e r a l M a t h e m a t i c s
Experience
Prelection
Activity 22

Concept Notes
Think of a logarithm of 𝑥 to the base 𝑏 (denoted by 𝑙𝑜𝑔𝑏 𝑥 as the exponent of 𝑏 that gives 𝑥.

EXAMPLE 1

EXAMPLE 2
Without using a calculator, find the value of the following logarithmic expression.

Solution

112 | G e n e r a l M a t h e m a t i c s
113 | G e n e r a l M a t h e m a t i c s
Representation of Real-Life Situations that use Logarithms

EXAMPLE 4
Suppose that an earthquake released approximately 1012 joules of energy.

114 | G e n e r a l M a t h e m a t i c s
B. Sample Application: The decibel

EXAMPLE 5
The decibel level of sound in a quiet office is 106 𝑤𝑎𝑡𝑡𝑠/𝑚2

Question: Answer:
10−6
(a) D = log −12 = 10log106 Hence by definition, log 106 is the
10
(a) What is the corresponding exponent by which 10 must be raised to obtained 106 , then log
sound intensity in decibels? 106 = 6,
Thus, D = 10(6) = 60 decibels.
(b) How much more intense is −6
this sound than the least (b) This sound is 10−12 = 106 = 100,000 times more intense than the
10
audible sound a human can least audible sound a human can hear.
hear?

115 | G e n e r a l M a t h e m a t i c s
Reflection
Questions:
1. What skills were you able to utilize that you can relate from this lesson?
______________________________________________________________________________
______________________________________________________________________________
______________________________________________________________________________

2. In your current situation, is there something that you want to change or convert for the better?
______________________________________________________________________________
______________________________________________________________________________
______________________________________________________________________________

3. We have our moments of ups and downs in our lives. Which part in your life were you able to
turn your down moments into something positive or use it as a way to transform into something
better?
______________________________________________________________________________
______________________________________________________________________________
______________________________________________________________________________
______________________________________________________________________________

Action
Activity 23
Now you have known that logarithmic function and exponential function are inverses of each
other, think one real life scenario where you can apply this concept of logarithmic and exponential
function in your daily life. Then draw a diagram which represent the situation and provide a brief
explanation.

116 | G e n e r a l M a t h e m a t i c s
Evaluation
Test I.
Directions: Answer the following correctly. Each item is equivalent to 2 points.

1. Write the following exponential form to logarithmic form.


𝟏 𝟑
a. 𝟓−𝟑 = b. (𝟖𝟏)𝟒 = 𝟐𝟕
𝟏𝟐𝟓

2. Write the logarithmic form to exponential form


a. 𝒍𝒐𝒈𝟑 𝒙 = 𝟏𝟎 b. 𝒍𝒐𝒈𝒄 𝒕 = 𝟗

𝟏 𝟏𝟔
a. 𝒍𝒐𝒈𝟐 b. 𝒍𝒐𝒈𝟐
𝟖 𝟑
𝟖𝟏

Test II
Directions: Solve the following word problem completely. Each item is equivalent to 5 points.

1. If an earthquake has a magnitude of 4 in the Richer scale, what is the size of the shockwave?

2. How many years will it take an investment of 25,000𝑝ℎ𝑝 to double in the value if interest is
compounded quarterly at the rate of 10%? (note that logarithmic function is an inverse of exponential
function)

117 | G e n e r a l M a t h e m a t i c s
Lesson 12 Basic Properties and Laws of Logarithms
LESSON PRE-ASSESSMENT
Directions: Determine whether the following equations below applies the basic properties and laws of
logarithms. Write TRUE if the equation is correct and FALSE if otherwise. Each item is equivalent to 1
point.

___________ 1. log(𝑏 − 𝑑 ) = log 𝑏 − log 𝑑

___________ 2. log 𝑥 𝑥 𝑥 = 𝑥

___________ 3. log(𝑥 ∙ 𝑦)2 = 2 log 𝑥 + 2 log 𝑦

32
___________ 4. log 2 = log 2 32 + log 2 4
4

___________ 5. 8log8 3 = 3

Context
Learning Competencies
At the end of the lesson, the learners will be able to:
✓ illustrate the basic properties of algorithms.
✓ illustrate the laws of logarithms.

Value Integration: Character

118 | G e n e r a l M a t h e m a t i c s
Experience
Prelection
Activity 24: Log Out!
Directions: Identify whether the given equation inside each box is NOT TRUE. Mark them by drawing a
big X mark on the box.

5
𝑙𝑜𝑔 = 𝑙𝑜𝑔5 − 𝑙𝑜𝑔6 𝑙𝑜𝑔(3 ∙ 8) = 𝑙𝑜𝑔 3 + 𝑙𝑜𝑔 8 𝑙𝑜𝑔5 52 = 25
6

1 𝑢3
𝑙𝑜𝑔2 (5 + 2) = 𝑙𝑜𝑔2 5 + 𝑙𝑜𝑔2 2 𝑙𝑜𝑔4 = −3 𝑙𝑜𝑔 = 3 𝑙𝑜𝑔 𝑢 − 𝑙𝑜𝑔 𝑤
64 𝑤

𝑎 𝑙𝑜𝑔3 5
𝑙𝑜𝑔 = log 𝑎 − 4 𝑙𝑜𝑔𝑏 𝑙𝑜𝑔3 (9 − 7) = 𝑙𝑜𝑔2 (5 − 2) = 𝑙𝑜𝑔2 5 − 𝑙𝑜𝑔2 2
𝑏4 𝑙𝑜𝑔3 2

Concept Notes

119 | G e n e r a l M a t h e m a t i c s
PROPERTY EXPLANATION EXAMPLE

Given: 𝑙𝑜𝑔 1
𝑙𝑜𝑔𝑏 1 is the exponent of b that
(1) 𝒍𝒐𝒈𝒃 𝟏 = 0 gives 1; in other words, 𝑏 ? = 1.
Solution:
This exponent must be 0.
𝑙𝑜𝑔 1 = 0

Given: 𝑙𝑜𝑔 10
𝑙𝑜𝑔𝑏 𝑏 𝑥 is the exponent of b that
(2) 𝒍𝒐𝒈𝒃 𝒃𝒙 = 𝒙 gives 𝑏 𝑥 , Clearly, the exponent
Solution:
must be x.
𝑙𝑜𝑔 10 = 𝑙𝑜𝑔10 101 = 1

𝑙𝑜𝑔𝑏 𝑥 is the exponent of b that Given: 5𝑙𝑜𝑔5 2


(3) If x >0,
gives x, So if we make it the
exponent of b, then we should Solution:
then 𝒃𝒍𝒐𝒈 𝒃𝒙 = x
get x. 5𝑙𝑜𝑔5 2 =2

EXAMPLE
LAW PROOF
(from the activity)
Let r = 𝑙𝑜𝑔𝑏 u and s = 𝑙𝑜𝑔𝑏 u .
Then u = 𝑏 𝑟 and 𝜐 = 𝑏 8 .

𝑙𝑜𝑔𝑏 (u𝜐) = 𝑙𝑜𝑔𝑏 (𝑏 𝑟 𝑏 8 )


𝒍𝒐𝒈𝒃 (u𝝊) = 𝒍𝒐𝒈𝒃 u + 𝒍𝒐𝒈𝒃 𝝊 ⟹ 𝑙𝑜𝑔𝑏 (u𝜐) = 𝑙𝑜𝑔𝑏 (𝑏 𝑟 𝑏 8 ) 𝑙𝑜𝑔7 (73 ∙ 78 ) = 𝑙𝑜𝑔7 73 + 𝑙𝑜𝑔7 78
⟹ 𝑙𝑜𝑔𝑏 (u𝜐) = r + 8

⟹ 𝑙𝑜𝑔𝑏 (u𝜐) = 𝑙𝑜𝑔𝑏 u =𝑙𝑜𝑔𝑏 𝜐

120 | G e n e r a l M a t h e m a t i c s
49
𝑙𝑜𝑔7 ( ) = 𝑙𝑜𝑔7 49 − 𝑙𝑜𝑔7 7
𝒖 The proof of the second law is 7
𝒍𝒐𝒈𝒃 ( ) = 𝒍𝒐𝒈𝒃 u - 𝒍𝒐𝒈𝒃 𝝊
𝒗 similar to the one above.
24
𝑙𝑜𝑔2 ( ) = 𝑙𝑜𝑔2 24 - 𝑙𝑜𝑔2 210
210

Let r =𝑙𝑜𝑔𝑏 .
Then u = 𝑏 𝑟 and 𝑢𝑛 = 𝑏 𝑟𝑛

𝑢𝑛 = 𝑏 𝑟𝑛 𝑙𝑜𝑔7 75 = 5 ∙ 𝑙𝑜𝑔7 7
𝒍𝒐𝒈𝒃 (𝒖𝒏 ) = 𝒏𝒍𝒐𝒈𝒃 u ⟹ 𝑙𝑜𝑔𝑏 (𝑢𝑛 ) =
𝑙𝑜𝑔𝑏 (𝑏 𝑟𝑛 ) 𝑙𝑜𝑔3 (27 ∙ 81) = 𝑙𝑜𝑔3 27 + 𝑙𝑜𝑔3 81
⟹ 𝑙𝑜𝑔𝑏 (𝑢𝑛 ) = rn
⟹ 𝑙𝑜𝑔𝑏 (𝑢𝑛 ) = n
𝑙𝑜𝑔𝑏 u

Example 1
Use the properties of logarithms to expand each expression in terms of the logarithms of the factors.
Assume each factor is positive.

121 | G e n e r a l M a t h e m a t i c s
Example 2
Use the change of base formula to rewrite the following logarithmic expressions to the indicated base,
and then compute the value.

Guided Practice
1. Find the value of the following logarithmic expressions.

(a) 𝑙𝑜𝑔7 49

(b) 𝑙𝑜𝑔27 3

(c) ln e

2. Use the properties of logarithm to expand the expressions as a sum, difference, or multiple of
logarithms. Simplify.

𝑥3
a. log ( )
2

b. ln (2𝑒)2

c. 𝑙𝑜𝑔4 (16a)

122 | G e n e r a l M a t h e m a t i c s
Reflection
Questions:
1. How did you find the activity?
______________________________________________________________________________
______________________________________________________________________________
______________________________________________________________________________

2. Which among the dares did you enjoy the most and why?
______________________________________________________________________________
______________________________________________________________________________
______________________________________________________________________________

3. In life’s decision making, there are pros and cons. Recall a particular situation in your life where
you were able to make the cons into a seed towards a better transformation in your life.
______________________________________________________________________________
______________________________________________________________________________
______________________________________________________________________________

Action
Activity 25
You know that logarithmic expressions have certain parts and characteristics. Consider yourself as a
logarithmic expression with all the uniqueness, attributes, and characteristics then applying the
concept of change of base formula, was there a part in your life that you need to change for the
better so that you can see your growth and worth as a person? Name and list these things with
explanation.

123 | G e n e r a l M a t h e m a t i c s
Evaluation
Test I.
Direction: Find the values of the given logarithms. Each item is equivalent to 2 points.

a. 𝒍𝒐𝒈𝟑 𝟕𝟐𝟗 b. 𝒍𝒐𝒈𝟕𝟐𝟗 𝟕𝟐𝟗

c. 𝒍𝒐𝒈𝟖 𝟖𝟑 d. 𝒃𝒍𝒐𝒈 𝒃𝟕

Test II.
Direction: Use the properties of logarithm to expand the expression as a sum, difference, or multiple of
logarithms. Each item is equivalent to 3 points.

𝒙
a. 𝒍𝒐𝒈 𝟓 𝟔
𝟑 b. 𝒍𝒐𝒈 ( )
𝟕

𝒂
c. 𝒍𝒐𝒈 (𝟒. 𝟑𝟓 ) d. 𝒍𝒐𝒈
𝒃𝟑

e. 𝒍𝒐𝒈 (𝒂. 𝒃. 𝒄𝟑 ) 𝟓
e. 𝒍𝒐𝒈 ξ𝒂. 𝒃. 𝒄

124 | G e n e r a l M a t h e m a t i c s
Test III.
Use the change of base formula to rewrite the following logarithmic expressions to the indicated base,
and then compute the value. Each item is equivalent to 3 points each.

1. 𝒍𝒐𝒈𝟔𝟒 𝟏𝟔 (change to base 2)

2. 𝒍𝒐𝒈𝟏𝟎𝟐𝟒 𝟔𝟒 (change to base 4)

𝟏
3. 𝒍𝒐𝒈𝟐𝟓 (change to base 5)
𝟔𝟐𝟓

125 | G e n e r a l M a t h e m a t i c s
Lesson 13 Solving Logarithmic Functions, Equations, and Inequalities
LESSON PRE-ASSESSMENT
Direction: Identify whether the given is a logarithmic function, logarithmic equality, logarithmic
inequality, or none. Write your answer on the space provided. Each item is equivalent to 1 point.

_____________1. log 4 (𝑥 + 1)

_____________2. log 4 = 2 − log(𝑥 + 5)

_____________3. log(𝑥 + 7) − 1 > log(𝑥 − 2)

_____________4. 𝑥 + 2

_____________5. 2𝑥 − 1 = 3

Context
Learning Competencies
At the end of the lesson, the learners will be able to:
✓ distinguish logarithmic function, logarithmic equation, and logarithmic inequalities;
✓ solve logarithmic equations and inequalities; and
✓ solve problems involving logarithmic equations and inequalities.

Value Integration: Competence

126 | G e n e r a l M a t h e m a t i c s
Experience
Prelection
Activity 26
Direction: Write your ideas about how A, B, and C are similar and how they are different.

Concept Notes
The definitions of Logarithmic equations, inequalities and functions are shown below.
Logarithmic Equation Logarithmic Inequality Logarithmic Function

An equation involving An inequality involving Function of the form f(x)


Definition
logarithms. logarithms. = 𝑙𝑜𝑔𝑏𝑥 (b > 0, 𝑏 ≠ 1).

Example 𝑙𝑜𝑔𝑥 2 = 4 ln 𝑥 2 > (ln 𝑥)2 g(x) = 𝑙𝑜𝑔3 𝑥

127 | G e n e r a l M a t h e m a t i c s
Solving logarithmic equations
Some strategies for solving logarithmic equations
• Rewriting to exponential form
• Using logarithmic properties
• Applying the One-to-One property of logarithmic functions, as stated below

One-to-One property of Logarithmic Functions


For any logarithmic function f(x) = 𝑙𝑜𝑔𝑏 𝑥, if 𝑙𝑜𝑔𝑏 𝑢 = 𝑙𝑜𝑔𝑏 𝜐 = then u = 𝜐.

Another useful property is the Zero Factor Property:


If ab = 0, then a = 0 or b = 0.

EXAMPLE 1
Find the value of x in the following equations.
a. 𝑙𝑜𝑔4 (2𝑥) = 𝑙𝑜𝑔4 10
b. 𝑙𝑜𝑔2 (x+1) + 𝑙𝑜𝑔2 (x-1) = 3

Solution

a. 𝑙𝑜𝑔4 (2𝑥) = 𝑙𝑜𝑔4 10


2x = 10 (one-to-one property)
x=5
Check: 5 is a solution since 𝑙𝑜𝑔4 (2∙ 5) = 𝑙𝑜𝑔4 (10).

128 | G e n e r a l M a t h e m a t i c s
b. 𝑙𝑜𝑔2 (x+1) + 𝑙𝑜𝑔2 (x-1) = 3
𝑙𝑜𝑔2 [(𝑥 + 1)(𝑥 − 1)] = 3 (using the property 𝑙𝑜𝑔𝑏 u + 𝑙𝑜𝑔𝑏 (𝑢𝜐))

(x+1)(x-1) = 23 q (Note: Zero Factor Property cannot be used yet)


𝑥2 - 1 = 8 (multiplication of two binomials)
𝑥2 - 9 = 0
(x+3)(x-3) = 0 (factorization using 𝑎2 − 𝑏 2 = (a + b)(a – b)
x = -3,3 (Zero factor property)
Check: 3 is a solution since 𝑙𝑜𝑔2 (3 + 1) and 𝑙𝑜𝑔2 (31) are defined.
however, -3 is not a solution since 𝑙𝑜𝑔2 (3 + 1) = 𝑙𝑜𝑔2 (2) is not defined.
Solution: x = 3 only.

Solving logarithmic inequalities


Property of Logarithmic Inequalities
Given the logarithmic expression 𝑙𝑜𝑔𝑏 𝑥,

If 0 < 𝑏 < 1, then 𝑥1 < 𝑥2 if and only if 𝑙𝑜𝑔𝑏 𝑥1 > 𝑙𝑜𝑔𝑏 𝑥2 .

If b > 1, 𝑡ℎ𝑒𝑛 𝑥1 < 𝑥2 𝑖𝑓 𝑎𝑛𝑑 𝑜𝑛𝑙𝑦 𝑖𝑓 𝑙𝑜𝑔𝑏 𝑥1 < 𝑙𝑜𝑔𝑏 𝑥2 .

EXAMPLE 2
a. 𝑙𝑜𝑔3 (2x – 1) > 𝑙𝑜𝑔3 (x + 2)

Solution
a. 𝑙𝑜𝑔3 (2x – 1) > 𝑙𝑜𝑔3 (x + 2)
Step 1: Ensure that the logarithms are defined.
Then 2x – 1 > 0 and x + 2 > 0 must be satisfied.
1 1
2x – 1 > 0 implies x > (if x > , then x is surely greater than -2).
2 2

Step 2: Ensure that the inequality is satisfied.


The base 3 is greater than 1.
Thus, since 𝑙𝑜𝑔3 (2x + 1) > 𝑙𝑜𝑔3 (x + 2), then:
2x – 1 > x + 2
x>3 (subtract x from both sides; add 1 to both sides)
∴x>3
Hence, the solution is (3, + ∞).

129 | G e n e r a l M a t h e m a t i c s
Guided Practice
1. Solve the following logarithmic equations.
a. 𝑙𝑜𝑔5 (𝑥 − 1) + 𝑙𝑜𝑔5 (𝑥 + 3) - 1 = 0

b. 𝑙𝑜𝑔3 x + 𝑙𝑜𝑔3 (x + 2) =1

2. Solve the following logarithmic inequalities.


a. ln x > 1 (Note: e ≈ 2.7183)

130 | G e n e r a l M a t h e m a t i c s
Reflection
Questions:
1. How did you find the activity?
_______________________________________________________________________________
_______________________________________________________________________________
_______________________________________________________________________________

2. Which among the activity did you enjoy the most and why?
_______________________________________________________________________________
_______________________________________________________________________________
_______________________________________________________________________________

3. Which of the activity did you like least and why?


_______________________________________________________________________________
_______________________________________________________________________________
_______________________________________________________________________________

Action
Activity 27
Applying the concepts of equations and inequalities in real life, there are many concerns in our
society where inequality or lack of evenness had been observed. Identify one issue or concern then
explain what you can do or solve it in order to attain equality.

131 | G e n e r a l M a t h e m a t i c s
Evaluation
Test I.
Directions: Solve the following logarithmic equations and inequalities. Show you complete solution on
the space provided for. Each item is equivalent to 5 points.

1. 𝒍𝒐𝒈𝟏𝟎 (𝟑𝒙 + 𝟓) = 𝟐 2. 𝒍𝒐𝒈𝟑 (𝟐𝒙 − 𝒙) = 𝟑

3. 𝒍𝒐𝒈(𝟑𝒙 − 𝟏) > 𝒍𝒐𝒈 (𝟒 − 𝒙) 4. 𝒍𝒐𝒈𝟐 𝒙 < 𝒍𝒐𝒈𝟐 (𝒙 + 𝟒)

132 | G e n e r a l M a t h e m a t i c s
Lesson 14 Graphs of Logarithmic Function
LESSON PRE-ASSESSMENT
Direction: Complete the table of values for the given 𝒚 = 𝒍𝒐𝒈𝟐 𝒙

𝟏 𝟏 𝟏
𝒙 𝟏 𝟐 𝟑 𝟒
𝟖 𝟒 𝟐
𝒚

Context
Learning Competencies
At the end of the lesson, the learners will be able to:
✓ represent a logarithmic function through its table of values, graphs, and equation;
✓ find the domain and rage of logarithmic function;
✓ determine the intercepts, zeroes, and asymptotes of logarithmic function; and
✓ graph logarithmic functions.

Value Integration: Competence

Experience
Concept Notes
Representing the logarithmic function (base greater than 1) through its equation, table of values and
graph

EXAMPLE 1
Sketch the graph of y = 𝑙𝑜𝑔2 x

Solution
Step 1: Construct a table of values of ordered pairs for the given function. A table of values for
y = 𝑙𝑜𝑔2 x is as follows:

1 1 1 1
x 1 2 4 8
16 8 4 2
y -4 -3 -2 -1 0 1 2 3

133 | G e n e r a l M a t h e m a t i c s
Step 2: Plot the points found in the table, and connect them using a smooth curve.

It can be observed that the function is defined only for 𝑥 > 0. The function is strictly increasing, and
attains all real values. As 𝑥 approaches 0 from the right, the function decreases without bound such
that the line 𝑥 = 0 is the vertical asymptote.

EXAMPLE 2
Sketch the graph of y = 𝑙𝑜𝑔1 𝑥
2

Solution
Step 1: Construct a table of values of ordered pairs for the given function. A table of values for
y = 𝑙𝑜𝑔1 𝑥 is as follows:
2
x 1 1 1 1 1 2 4 8
16 8 4 2
y 4 3 2 1 0 -1 -2 -3

Step 2: Plot the points found in the table, and connect them using a smooth
curve.

It can be observed that the function is defined only for 𝑥 > 0. The function is strictly decreasing, and
attains all real values. As 𝑥 approaches 0 from the right, the function increases without bound such that
the line 𝑥 = 0 is the vertical asymptote.

134 | G e n e r a l M a t h e m a t i c s
Relationship between the Graphs of Logarithmic and Exponential Functions.
Since logarithmic and exponential functions are inverses of each other, their graphs are reflections of
each other about the line 𝑦 = 𝑥, as shown below.

EXAMPLE 3
Sketch the graph of 𝑦 = 2𝑙𝑜𝑔2 𝑥. Determine the domain, range, vertical asymptote, x-intercept, and
zero.

Solution
Step 1: The graph of 𝑦 = 2𝑙𝑜𝑔2 𝑥 can be obtained from the graph of 𝑦 = 𝑙𝑜𝑔2 𝑥 by multiplying each
𝑦 − 𝑐𝑜𝑜𝑟𝑑𝑖𝑛𝑎𝑡𝑒 by 2, as the following table of signs shows.
Construct a table of values of ordered pairs for the given function.
A table of values for 𝑦 = 2𝑙𝑜𝑔2 𝑥 is as follows:

135 | G e n e r a l M a t h e m a t i c s
Step 2: Plot the points found in the table, and connect them using a smooth curve.

Guided Practice
Activity 28: Log Tank!
Direction: Complete the graphic organizer below by providing the necessary answers to the
question/statement that is being asked. The function is given below.

136 | G e n e r a l M a t h e m a t i c s
Reflection
Questions:
1. How did you find the activity?
_______________________________________________________________________________
_______________________________________________________________________________
___________________________________________________________________________.

2. Which among the activity did you enjoy the most and why?
_______________________________________________________________________________
_______________________________________________________________________________
___________________________________________________________________________.

3. Which of the activity did you like least and why?


_______________________________________________________________________________
_______________________________________________________________________________
__________________________________________________________________________

Action
Activity 29
Identify one real life application of logarithmic function then draw a diagram that represents it, and
provide a brief explanation.

137 | G e n e r a l M a t h e m a t i c s
Evaluation
Direction: Sketch the graph of given function then identify the y-intercept, vertical asymptote, domain,
and range.

Given: 𝒍𝒐𝒈𝟏 (𝒙 − 𝟏)
𝟑

Domain Range

Vertical asymptote y-intercept

Graph

138 | G e n e r a l M a t h e m a t i c s
UNIT 1 Answer Keys
UNIT PRE-ASSESSMENT

LESSON 1. Functions as Models


LESSON PRE-ASSESSMENT
1. TRUE 2. FALSE 3. FALSE 4. TRUE 5. TRUE

PRELECTION

139 | G e n e r a l M a t h e m a t i c s
GUIDED PRACTICE

140 | G e n e r a l M a t h e m a t i c s
LESSON 2. Evaluating a Function
LESSON PRE-ASSESSMENT
1. Possible answers:

• A correspondence of two sets wherein for every input there must be only one output.
• A relation with restriction that its mapping must not be a one-to-many relationship.
Any other answers that are similar or having the same thought can be marked as correct.

2. a. Function

b. Function

3. a. 𝑓(0) = −5

b. 𝑓(𝑎) = 𝑎 − 5

4. a. Domain: {0,2,4} Range: {−1, −5, −9}

b. Domain: {−4, −1, 3, 6, 9} Range: {1, 9, 16, 36, 81}

5. a. Function

b. Function

c. Not a Function

d. Not a Function

GUIDED PRACTICE
2. 𝑓(1.5) = 8

3. 𝑓(0) = 2

4. 𝑓(2𝑥) = 4𝑥 2 + 10𝑥 + 3

1
5. 𝑓(𝑎) =
2

6. 𝑓(𝑎 + 𝑏) = 𝑎2 + 2𝑎𝑏 + 𝑏 2 + 5

141 | G e n e r a l M a t h e m a t i c s
LESSON 3. Operations on Functions
LESSON PRE-ASSESSMENT
a. (𝑓 + 𝑔)(𝑥) = −2𝑥 − 2

b. (𝑓 − 𝑔)(𝑥) = 6𝑥 + 8

c. (𝑓 ∙ 𝑔)(𝑥) = −8𝑥 2 − 22𝑥 − 15

2𝑥+3
d. (𝑓/𝑔)(𝑥) =
−4𝑥−5

e. (𝑓 ∘ 𝑔)(𝑥) = −8𝑥 − 7 𝑜𝑟 − (8𝑥 + 7)

f. (𝑔 ∘ 𝑓)(𝑥) = −8𝑥 − 17 𝑜𝑟 − (8𝑥 + 17)

PRELECTION
Answer: 9 − 8 + (7 − 6 − 1) × 3 = 1

GUIDED PRACTICE
(𝑓 + 𝑔)(𝑥) = 𝑥 2 + 3𝑥 − 2

(𝑓 − 𝑔)(𝑥) = −𝑥 2 + 3𝑥 − 2

(𝑓 ∙ 𝑔)(𝑥) = 3𝑥 3 + 2𝑥 2

3𝑥 − 2
(𝑓/𝑔)(𝑥) =
𝑥2

(𝑓 ∘ 𝑔)(𝑥) = 3𝑥 2 − 2

(𝑔 ∘ 𝑓)(𝑥) = 9𝑥 2 − 12𝑥 + 4

(𝑓 ∘ 𝑓)(𝑥) = 9𝑥 − 8

(𝑔 ∘ 𝑔)(𝑥) = 𝑥 4

142 | G e n e r a l M a t h e m a t i c s
LESSON 4. Rational Function
LESSON PRE-ASSESSMENT
1 1 1
1. + = , where t is the length of time
8 𝑡 5
5 5 4
2. − = , where v is the speed
𝑣 𝑣+10 3
5𝑥 2
3. = −10where x is the unknown integer
3𝑥+4
222 66
4. = where x is the rate of the current
24+𝑥 24−𝑥

PRELECTION
Expected Answers: ½, 1:2, ½ = 2/4, equivalent fractions

GUIDED PRACTICE

1. Step 1: Let n be the number of members the 2. Step 1: Let x = rate of the current
institution has n + 30 if we add an additional
30 members 2+x = rate along the current
2-x = rate against the current
2000 2000
Step 2: − = 150 32 16
𝑛 𝑛+30
Step 2: =
2+𝑥 2−𝑥

3. Step 1: Let x be the value of the unknown

1 33
Step 2: 2 (𝑥 + ) =
𝑥 16

2𝑥 2 +2 33
Answer: =
𝑥 16

LESSON 5. Solving Rational Equation and Inequality


LESSON PRE-ASSESSMENT
C 1. Rational Equation
D 2. Rational Equation
B 3. Rational Inequality
A 4. Rational Function

PRELECTION
a. (𝑥 + 2)(𝑥 − 2) e. (2𝑥 − 5)(2𝑥 + 5)
b. 10𝑥 f. 5(𝑎 − 4)
c. 2(3𝑥 + 2) g. (𝑥 − 6)(𝑥 + 6)
d. 4

143 | G e n e r a l M a t h e m a t i c s
GUIDED PRACTICE
3𝑥+𝑥+1
1. =4
𝑥(𝑥+1)
4𝑥 + 1 = 4𝑥 2 + 4𝑥
4𝑥 2 − 1 = 0
1 1
𝑥1 = , 𝑥2 = −
2 2

3 1
1. + ≤4
𝑥+1 𝑥

Step 1:
4𝑥 + 1
−4 ≤ 0
𝑥(𝑥 + 1)

4𝑥 + 1
−4 ≤ 0
𝑥(𝑥 + 1)
4𝑥 + 1 − [4(𝑥 2 + 𝑥)]
≤0
𝑥(𝑥 + 1)
1 − 4𝑥 2
≤0
𝑥(𝑥 + 1)

1 1
𝑥1 = , 𝑥2 = −
2 2
Step 2:

𝑥(𝑥 + 1) =0
𝑥1 = 0, 𝑥2 = −1

Step 3:

(−∞, −1)(−1, −1/2)(−1/2,0)(0,1/2)(1/2, ∞)

Factor/ (−∞, −1) (−1, −1 (−1/2,0) (0,1/2) (1/2, ∞)


Intervals /2)
1 − 4𝑥 2 − − + + −
𝑥(𝑥 + 1) + − − + +
1 − 4𝑥 2 − + − + −
𝑥(𝑥 + 1)
Step 4:
Step 5:
(−∞, −1) ∪ −1/2,0) ∪ 1/2, ∞)

144 | G e n e r a l M a t h e m a t i c s
LESSON 6. Graphing Rational Function
LESSON PRE-ASSESSMENT

GUIDED PRACTICE

145 | G e n e r a l M a t h e m a t i c s
LESSON 7. One-to-One and Inverse Function
LESSON PRE-ASSESSMENT
1. d
2. c
3. a
4. c

GUIDED PRACTICE

146 | G e n e r a l M a t h e m a t i c s
LESSON 8. Representing Real-Life Situations Using Exponential Functions
LESSON PRE-ASSESSMENT
(1) C (2) A (3) A (4) B (5) D

GUIDED PRACTICE
(1) a. 𝐴 = 50,000(1.044)𝑡 b. P100,000
𝑥ൗ
1 250
(2) a. 𝑦 = 100 ( ) b. 25g
2

100ൗ
(3) a. 𝑦 = 1,000(3) 80 b. 3,948

LESSON 9. Solving Exponential Functions, Equations, and Inequalities


LESSON PRE-ASSESSMENT
(1) None (2) Exponential Function
(3) Exponential Function (4) Exponential Equation
(5) Exponential Inequalities

PRELECTION
Peter is correct, John is not.

GUIDED PRACTICE
(1) Equation : (a), (c), (d), (f) Inequalities : (b), (e), (g)
8 29
(2) a. 𝑥 = b. [ , +∞)
3 6

147 | G e n e r a l M a t h e m a t i c s
LESSON 10. Graphing Exponential Functions
LESSON PRE-ASSESSMENT
(1) b (2) c (3) d (4) e (5) a

GUIDED PRACTICE

LESSON 11. Logarithmic Function


LESSON PRE-ASSESSMENT
1. d 2. b 3. a 4. d 5. b

PRELECTION
a.) 4 b.) -2 c.) 0 d.) 5

GUIDED PRACTICE
1
1. a.) 𝑙𝑜𝑔3 243 = 5 b.) 𝑙𝑜𝑔6 ( ) = −3
216
1
2. a.) 𝑙𝑜𝑔 100 = 2 b.) 𝑙𝑜𝑔6 ( ) = −3
216
3. a.) 𝑒𝑦 =3 b.) 10−3 = 0.001

148 | G e n e r a l M a t h e m a t i c s
LESSON 12. Basic Properties and Laws of Logarithms
LESSON PRE-ASSESSMENT
1. FALSE 2. TRUE 3. TRUE 4. FALSE 5. TRUE

PRELECTION(Activity)
Logarithm with the same answer Answer
1. 𝑙𝑜𝑔7 (73 ∙ 78 ) and 𝑙𝑜𝑔7 73 + 𝑙𝑜𝑔7 78 11
49
2. 𝑙𝑜𝑔7 ( ) and 𝑙𝑜𝑔7 49 - 𝑙𝑜𝑔7 7 1
7
24
3.𝑙𝑜𝑔2 ( 10 ) and 𝑙𝑜𝑔2 24 - 𝑙𝑜𝑔2 210 -6
2
4. 𝑙𝑜𝑔3 (27 ∙ 81) and 𝑙𝑜𝑔3 27 + 𝑙𝑜𝑔3 81 7

GUIDED PRACTICE
1
1. (a) 1 (b) (c) 1
3
2. (a) 3 log 𝑥 − 𝑙𝑜𝑔 2 (b) 2 ln 2 + 2 (c) 2 𝑙𝑜𝑔4 𝑎
𝑙𝑜𝑔 4 2 𝑙𝑜𝑔 3 1
3. (a) 𝑙𝑜𝑔6 4 = 2 = (b) 𝑙𝑜𝑔1 3 = 3 1 =
𝑙𝑜𝑔2 6 𝑙𝑜𝑔2 6 9 𝑙𝑜𝑔3 −2
9

LESSON 13. Solving Logarithmic Functions, Equations, and Inequalities


LESSON PRE-ASSESSMENT
1. none of these
2. logarithmic equation
3. logarithmic inequality
4. none of these
5. logarithmic function

PRELECTION
1
𝑥 = −2 and 𝑥 = 2 (b) (c) 2
16

149 | G e n e r a l M a t h e m a t i c s
GUIDED PRACTICE

1. (a) 𝑙𝑜𝑔5 (𝑥 − 1) + 𝑙𝑜𝑔5 (𝑥 + 3) − 1 = 0 Check : 2 is a solution since 𝑙𝑜𝑔5 (2 − 1) + 𝑙𝑜𝑔5 (2 + 3) − 1 =


𝑙𝑜𝑔5 (𝑥 − 1) + 𝑙𝑜𝑔5 (𝑥 + 3) = 1 0
𝑙𝑜𝑔5 [(𝑥 − 1)(𝑥 + 3)] = 1
(𝑥 − 1)(𝑥 + 3) = 51 𝑙𝑜𝑔5 (1) + 𝑙𝑜𝑔5 (5) − 1 = 0
𝑥 2 + 2𝑥 − 3 = 5 0 + 1 − 1 = 0
𝑥 2 + 2𝑥 − 3 − 5 = 0 0=0 hence it is defined
𝑥 2 + 2𝑥 − 8 = 0 However -4 is not a solution since 𝑙𝑜𝑔5 (−4 − 1) = 𝑙𝑜𝑔5 (−5)
(𝑥 + 4)(𝑥 − 2) = 0
𝑥+4=0 𝑥−2 =0
is not defined
𝑥 = −4 𝑥=2 Solution: 𝒙 = 𝟐 only

(b) 𝑥 = −4 𝑙𝑜𝑔
𝑎𝑛𝑑3 𝑥x +=𝑙𝑜𝑔
2 3 (x + 2) =1 Check : 1 is a solution since 𝑙𝑜𝑔3 1 + 𝑙𝑜𝑔3 (1 + 2) = 1
𝑙𝑜𝑔3 [(𝑥)(𝑥 + 2)] = 1 𝑙𝑜𝑔3 1 + 𝑙𝑜𝑔3 (3) = 1
𝑥(𝑥 + 2) = 31 0 + 1=1 Hence it is defined
𝑥 2 + 2𝑥 − 3 = 0 However -3 is not a solution since 𝑙𝑜𝑔3 − 3 is not
(𝑥 + 3)(𝑥 − 1) = 0 defined
Solution: 𝒙 = 𝟏 only
𝑥 = −3 and 𝑥 = 1
𝑥 = −4 𝑎𝑛𝑑 𝑥 = 2

𝑥 = −4 𝑎𝑛𝑑 𝑥 = 2
2.) −2 < log 𝑥 < 2
Step 1: Ensure that the logarithms are defined.
We first rewrite -2 and 2 as logarithms to base 10, which are log 10−2 and log 102 respectively, obtaining the
inequality:
log 10−2 < log 𝑥 < log 102
We split the compound inequality into two simple inequalities:
𝑙𝑜𝑔10−2 < log 𝑥 and log 𝑥 < 𝑙𝑜𝑔102
Since the base 10 is greater than 1, we simplify both inequalities as
10−2 < 𝑥 and 𝑥 < 102
1 1
Thus obtaining 100 < 𝑥 < 100, which automatically satisfies the condition in Step 1. Hence, the solution is (100 , 100)

𝑥 = −4 𝑎𝑛𝑑 𝑥 = 2

LESSON 14. Graphs of Logarithmic Function


LESSON PRE-ASSESSMENT
x 1 1 1 1 2 4 8
8 4 2
y -3 -2 -1 0 1 2 3

GUIDED PRACTICE
Table of Values: Possible answers
𝒙 1 3 9

𝒍𝒐𝒈𝟑 𝒙 0 1 2
𝒍𝒐𝒈𝟑 𝒙 − 𝟏 -1 0 1

150 | G e n e r a l M a t h e m a t i c s
Graph

Analysis:

(a) Domain: {𝑥|𝑥 ∈ 𝑅, 𝑥 > 0}


(b) Range: {𝑦|𝑦 ∈ 𝑅}
(c) Vertical asymptote: 𝑥 = 0
(d) x-intercept: (3,0)
(e) zero : 3

151 | G e n e r a l M a t h e m a t i c s
References

Define and write Piecewise functions | Intermediate algebra. (n.d.). Lumen Learning – Simple Book

Production.

Definition of function. (n.d.). Math is Fun. https://www.mathsisfun.com/definitions/function.html

Image retrieved from https://www.canon.co.uk/press-centre/press-releases/2019/01/smallest-five-ink-

single-function-printer/

https://courses.lumenlearning.com/intermediatealgebra/chapter/read-define-and-write-piecewise-

functions/

Piecewise functions. (n.d.). Math is Fun. https://www.mathsisfun.com/sets/functions-piecewise.html

Piramide, J.M., & et al., (2017)., Functions, investment &logic: keys to the real world: A

comprehensive worktext in general mathematics for senior high school. ClintKAMMS Corp.,

Mandaue City, Philippines.

Versosa, D.M., & et al., (2016)., Teaching guide for senior high school general mathematics.,

Commission on Higher Education., EC-TEC Commercial., Quezon City, Philippines.

152 | G e n e r a l M a t h e m a t i c s
Project Title: ROAD MAP TO BECOMING MEN AND
WOMEN WITH AND FOR OTHERS

G oal The learners can accurately construct mathematical models to represent real-life
situations using functions in their chosen field or profession in the future. Refer to this
link below:

https://www.youtube.com/watch?v=svFSKnmAlKQ

R ole As a researcher or developer or analyst of your company or institution or organization,


you are task to construct a research or a proposal using data from current events on how
to develop system, or how to increase profit or on how to market strategy.

A udience Stakeholders such as, employees, top management, investors and others who have
direct and indirect impact in the business operations. Administrators, Board of Director,
Engineer, Medical Practitioners.

S ituation In your chosen field or profession in the future you will be able to apply in real life
situation the knowledge you acquire. You are asked to conduct a research on how to
develop the system/model, or how to increase sales and profitability and improves
organizational performances such as costumer service.

P erforman Different mathematical models using functions in your chosen field or profession.
ce

153 | G e n e r a l M a t h e m a t i c s
The students will be graded according to the 4 C’s using a scale from 1 to 4 where 4 is
S tandard the highest point and 1 as the lowest.

Collaboration:
Each member of the group has a task to perform and are ask to collaborate with others
to accomplish the task. The students show effective troubleshooting skills in terms of
dealing with unforeseen circumstances. The leader of the group was able to facilitate
well the members in the group in doing and distributing the task.

Communication:
All members of the group articulated their ideas clearly. They were able to justify the
models presented clearly and constructively. Each member was able to answer the
questions asked by the audience. Articulate thoughts and ideas effectively using oral,
written, and nonverbal communication skills in presenting, dealing with others. Listen
effectively to the instructors and members of the group to decipher meaning, including
knowledge, values, attitudes, and intentions.

Critical Thinking and Problem Solving: The students can give justification on the models
created. Use various types of reasoning as appropriate to the situation. Analyze how
each part of the tasks interact with each other to come up with the final output or
product.

Creativity and Innovation: Students use a wide range of idea creation techniques (such
as brainstorming) to come up with the plan and project itself. The group gave
recommendations to improve the system of the school. Students develop, implement,
and communicate new ideas to the officials and school authorities effectively. Students
are open and responsive to new and diverse perspectives within the school and within
the group; incorporate group input and feedback into the work field.

_____________________________________________________________________________
___________________

5C’s

Competence - the model is detailed; the group shows knowledge and skills that enable
them to perform diverse tasks and carry out various responsibilities. Sound decision is
demonstrated. The students are beginning to further develop their full potentials, and is
continuously seeking opportunities to develop one’s personality.

154 | G e n e r a l M a t h e m a t i c s
GRASPS

Project Based Learning

Subject, Grade Level, Quarter

General Mathematics, Grade 11, First Quarter

Performance Standard
The student should be able to,
The learner is able to accurately construct mathematical models to represent real-life situations using
functions
Performance Task 1
Classifying relations and functions in real-life situations
Identify three (3) scenarios in real-life which are relations and another three (3) scenarios which are
functions

Performance Task 2
Identify common scenarios in the different learning environment and offices of the school
which involves functions
Final Product
Create a model of the identified scenarios (Performance task 2)

155 | G e n e r a l M a t h e m a t i c s
156 | G e n e r a l M a t h e m a t i c s

Potrebbero piacerti anche